Microbiology 221 Exam 3 MCQ's

Ace your homework & exams now with Quizwiz!

A major difference between the injected influenza vaccine and the "FluMist" nasal spray is that . . . A. "FluMist" doesn't have to be changed as often to compensate for antigenic drift B. the injected vaccine elicits mostly cellular immunity C. the injected vaccine is much more effective, although it is harder to make D. "FluMist" also blocks uncoating of the virus intracellularly E. "FluMist" induces active IgA immunity in a major portal of entry for the virus

"FluMist" induces active IgA immunity in a major portal of entry for the virus

To what do NK cells bind? A. MHC-II on APCs B. MHC-II on TC cells C. PAMPs on the surface of bacterial cells D. "Killer receptors" on any eukaryotic cell

"killer receptors" on any eukaryotic cell

Some antigens (T-independent) can activate B cells without the involvement of cellular immunity. Which of the following would still occur in response to these antigens? A. Memory B cells would be produced B. B cell receptors would bind to the antigen C. B cells would internalize the antigen D. TH cells would bind to the B cells' MHC2

B-cell receptors would bind to the antigen

The pathological structure indicated by the arrow in the throat of this 14-year- old immunocompetent patient is most likely caused by . . . A. cytokine-induced inflammation in response to an infection B. growth of a fungus on the tonsils of the patient C. a bacterial AB toxin that is damaging throat epithelial cells D. a virus that could have been prevented with the MMR vaccine E. a Type II hypersensitivity response triggered by immune mimicry

a bacterial AB toxin that is damaging throat epithelial cells

Which of the following can be killed by NK cells? A. Dendritic cells, but only after they have engulfed a pathogen B. Only foreign parasites circulating in the blood C. Only tumor cells D. Any cell that lacks MHC-I or has antibodies bound to it E. Any lymphocyte that is displaying a foreign antigen

any cell that lacks MHC-I or has antibodies bound to it

What type of cells do NK cells kill? A. any bacteria with PMPs (or other recognition signals) exposed B. B cells with B cell receptors whose Fab parts the NK cells recognize C. TC cells without B7 on their surface D. T cells that have phagocytized bacteria E. any cells without MHC1 (or its equivalent) on their surface

any cells without MHC1 (or its equivalent) on their surface

MHC Class II receptors . . . A. Are found only on phagocytes and B-cells B. Pick up antigen from the infected cell's cytoplasm C. Serve as binding receptors for TC cells D. bind to the CD8 receptor on T cells E. are found on T cells and used as transporters for cytokine secretion

are found only on phagocytes and B-cells

Injection of a patient with tetanus immune globulin is an example of . . . A. artificial, active immunity B. artificial, passive immunity C. natural, active immunity D. natural, passive immunity E. desensitization to overcome a Type I hypersensitivity

artificial, passive immunity

A TH cell is secreting interleukin-2 (IL-2). This means that . . . A. it will soon undergo apoptosis. B. it will now be able to ingest bacteria that bind to its TLR receptors. C. it is infected by a virus and has sensed the presence of double-stranded RNA. D. nearby TC cells will become activated once they bind to an antigen-MHC1 complex. E. an autoimmune disease is allowing the TH cell to recognize a self antigen.

nearby TC cells will become activated once they bind to an antigen-MHC1 complex

Which of the following can antibodies do to "fight" against both bacteria and viruses? A. neutralize receptor binding proteins B. lyse the virus or bacterium C. initiate the formation of the membrane attack complex D. bind to and inactivate toxins E. initiate antibody-dependent cellular cytoxicity (ADCC)

neutralize receptor binding proteins

A person with rheumatoid arthritis makes antibodies against the protein collagen, which is found in cartilage. These antibodies can cause a Type III hypersensitivity reaction. Which of the following best describes this reaction? A. NK cells attack cartilage that has antibodies bound to it. B. Mast cells bind the anti-collagen antibodies and release Type III mediators. C. Neutrophils recruited by complement peptides release inflammatory cytokines. D. IgE and IgG compete to form a complex that recruits TC cells and causes ADCC. E. The excess presence of collagen causes a strong T cell response

neutrophils recruited by complement peptides release inflammatory cytokines

What happens if you are exposed to an antigen but none of your circulating B cells has receptors that recognize it? A. You can produce new B cell receptors that may recognize this antigen. B. Your antibodies will still recognize it, even though your B cells don't. C. You should be vaccinated against it, because you will have no immunity against it. D. You will never be able to mount a humoral immune response against it. E. New B cells can be released from your bone marrow that may recognize it, even though no circulating B cells do

new B cells can be released from your bone marrow that may recognize it, even though no circulating B cells do

A male patient ingests a dose equal to the ID50 of E. coli that expresses the type P pilus. What would be the most likely outcome? A. Pneumonia B. Diarrhea C. Urinary tract infection D. An induced autoimmune disease E. No infection

no infection

Are we concerned about herd immunity levels when using passive immunizations? Why or why not? A. No. Passive immunization is too temporary for herd immunity to be important. B. Yes. If too few people are vaccinated, a disease can spread in a community. C. No. Herd immunity only applies to natural infections, not immunizations. D. Yes. If everyone is passively immunized, no one will get the disease. E. Passive immunity is a natural event; there is no such thing as "passive immunization."

no. passive immunization is too temporary for herd immunity to be important

Which part of this T cell receptor is produced only AFTER the T cell has bound an antigen? A. The part labeled "A" B. The parts labeled "B" C. The entire T cell receptor D. None of it. It is all produced BEFORE antigen binding. E. T cells don't bind antigens; only antibodies do that

none of it. it is all produced BEFORE antigen binding

The bacterium Legionella pneumophila causes an acute bacterial pneumonia with an incubation period of 3 days (not one we discussed in class). It can often be diagnosed only by serological tests. A patient with pneumonia arrives at a Dr's office for various tests, including a titer for anti-Legionella IgG. The titer is 1:32. No diagnosis is made, but the patient eventually recovers. The Dr still wants to know what caused the illness, though, so she runs another IgG titer. If the patient's pneumonia had been caused by Legionella, which of the following antibody titers might the Dr expect to find? A. 1:256 B. 1:64 C. 1:32 D. 1:8 E. zero, if all the bacteria are gone from the patient's body

1:256

In DiGeorge's Syndrome, the patient is born without a thymus. Such a patient obviously lacks some of his immune system. Which of the following best represents his antibody titer after initial and booster vaccination?

A

Which of the following B cells would become anergic? A. A B-cell that has a TH cell bound to its MHC B. A B-cell that has not bound to any antigen C. A B-cell that has a macrophage bound to its TLR D. A B-cell that fails to bind to an antigen during development in bone marrow E. A B-cell that is displaying an antigen to which no T-cell has bound

A B-cell that is displaying an antigen to which no T-cell has bound

A frequent cause of the common cold, this acid-resistant virus can be transmitted by the fecal-oral route as well as the respiratory route. A. Influenza B. Measles C. Rhinovirus D. Varicella E. Adenovirus

Adenovirus

Sam has a bad cold, with a runny nose and fever. He has not been closer than 1 meter to other people within the past week. Like 40% of students, Sam is not in the habit of washing his hands after using the toilet. Which of the following is the most likely cause of his cold? A. Streptococcus pneumoniae B. Mycoplasma pneumoniae C. Rhinovirus D. Influenza virus E. Adenovirus

Adenovirus

Upper respiratory infection whose symptoms may look like Strep throat. A. Adenovirus B. Rhinovirus C. Influenza D. Pertussis E. Respiratory syncytial virus

Adenovirus

Which of the following graphs best depicts the incidence (solid line) and prevalence (dotted line) of an acute emerging disease that is transmitted by respiratory droplets? A. A B. B C. C D. D E. E

B

In a T-dependent B cell response . . . A. multiple B cell receptors are crosslinked by antigen B. a B cell receptor must interact with a T cell receptor C. a B cell must endocytose an antigen and display it on MHC-II D. a T cell receptor must bind to CD8 on the B cell surface E. somatic recombination occurs, but not clonal selection

B cell must endocytose an antigen and display it on MHC-II

Which of the following steps occurs during the immune response to BOTH polysaccharide vaccines AND protein-conjugate vaccines? A. B cells engulf the vaccine antigen into an endocytic vesicle B. B cell receptors bind to polysaccharide components of vaccine antigens C. T cell receptors interact with the vaccine antigen presented on MHC-II D. Cytokines induce the formation of plasma cells and B memory cells E. Vaccine antigens are engulfed by macrophages or dendritic cells

B cell receptors bind to polysaccharide components of vaccine antigens

What happens during the process called "clonal deletion"? A. B cells that have bound to an antigen turn into plasma cells B. Variable genes are joined to diversity genes by joining segments to expand antibody diversity. C. B cells that have bound to a repeating antigen release surface receptors as antibodies D. B cells apoptose if their receptors recognize proteins present in bone marrow E. B cells that fail to recognize a T-cell receptor lose their ability to be activated

B cells apoptose if their receptors recognize proteins present in bone marrow

Which of the following is one way our immune system avoids producing antibodies that recognize self antigens? A. Somatic recombination is blocked if it produces self-recognizing B cells. B. B cells become anergic if they are not also stimulated by TH cells. C. B cells that fail to also bind TH cells in bone marrow apoptose. D. A B cell binds to MHC-II, which only displays foreign antigens. E. Such antibodies are produced, but they are opsonized and phagocytosed

B cells become anergic if they are not also stimulated by TH cells

A 7-year-old girl went to the Doctor with a slight fever and swollen lymph nodes on her upper arm and neck. She said she had been playing "dress up" with her cat 2 weeks ago, and the cat didn't like the game. Which of the following diagnoses was most likely? A. Candida albicans B. Bartonella henselae C. Sporothrix schenckii D. Epstein-Barr Virus E. Pasteurella multocida

Bartonella henselae

Which complement molecule is correctly matched with its function? A. C5a - chemokine signaling molecule B. C5b - part of a protease C. properdin - stimulates the inflammatory response D. C3b - lyses infected cells E. C3a - an opsonin that binds to bacteria

C5a - chemokine signaling molecule

A baby develops a thick white growth in his mouth and on his tongue. What microbial agent is probably responsible for this? A. Papillomavirus B. Staphylococcus aureus C. Corynebacterium diphtheriae D. Varicella-Zoster Virus E. Candida albicans

Candida albicans

Red skin rashes and white cheesy growth in moist areas of the body are caused by . . . A. Candida albicans B. Staphylococcus aureus C. Corynebacterium diphtheriae D. Bartonella henslae E. Pseudomonas aeruginosa

Candida albicans

Which of the following fungal diseases is NOT correctly matched with a risk factor? A. Histoplasmosis - aerosolized bird droppings B. Candidiasis - failure to be vaccinated against it C. Sporotrichosis - puncture injuries by plant thorns D. Coccidioidomycosis - immune system impairment (e.g. AIDS) E. All diseases are correctly matched with risk factors.

Candidiasis - failure to be vaccinated against it

Most of the time you don't take antibiotics for a sinus infection; however, if you are a child and have more than three cases of sinusitis caused by Haemophilus influenzae type b (Hib) within a few months, a doctor will likely prescribe antibiotics. What is the concern that prompts antibiotic use in this case? A. Antibodies produced against Hib can also damage your kidneys. B. Such prolonged sinusitis might develop into pneumonia. C. Chronic Hib infections can cause ear damage, and can even lead to meningitis. D. Antibiotics may be needed to disrupt the Hib biofilm that has formed in your lungs. E. If the Hib infection is during flu season, the Dr is concerned that you'll spread the flu.

Chronic Hib infections can cause ear damage, and can even lead to meningitis

This organism forms two different types of spores - arthrospores in the environment, and endospores in the host. There is a strong inflammatory response to the endospores, resulting in lung and tissue granuloma formation. What is the organism? A. Hantavirus B. Bacillus anthracis C. Histoplasma capsulatum D. Pasteurella multocida E. Coccidioides immitis

Coccidioides immitis

How are T cells involved in antibody production? A. They aren't - antibodies are produced by B cells without any help from T cells. B. Cytokines from TH cells cause proliferation and differentiation of B cells into plasma cells. C. T cells present the antigen that B cells recognize to begin secretion of antibodies. D. TC cells provide the "second signal" that tells B cells they have found a foreign antigen. E. Binding of both a B cell and a T cell to the same antigen results in B cell anergy

Cytokines from TH cells cause proliferation and differentiation of B cells into plasma cells.

An epidemic has struck summer camp! It began as a foodborne illness because they pay the cook minimum wage, but has become fecal-oral because the camp has only outdoor toilets and no running water to wash hands. What would the incidence graph for this disease look like? A. A B. B C. C D. D E. E

D

Typically, if a person's serum is diluted 1:4, it will still test positive for Legionella bacterial antigens. This is due to background exposure, and does not reflect an infection. In the following photos, Legionella antigen has been placed in the center well, and dilutions of a person's serum have been placed in outer wells as indicated (counterclockwise is more dilute, starting with 1:2 at the top). Which test indicates that a person has been infected with Legionella? A. B. C. D. E.

D

In the disease Lupus, autoantibodies are directed against A. DNA B. B cells C. Epithelial cell receptors D. The neurotransmitter acetylcholine E. Almost anything you touch or eat

DNA

In the disease lupus, your B cells recognize your own DNA, causing a type III hypersensitivity. How does this cause damage to your body? A. Your antibodies bind DNA on the surface of mast cells, which release histamine. B. The B cells then activate T cells, which mediate a cytotoxic response. C. The B cells produce antibodies that bind to DNA and inhibit DNA replication. D. DNA-Antibody complexes form and get stuck between cells, causing inflammation. E. DNA acts like a hapten to allow IgG to bind your cells and recruit NK cells.

DNA-Antibody complexes form and get stuck between cells, causing inflammation

A homeless man develops a chronic cough. When it gets so bad that he begins to cough up blood, he goes to a community clinic, where the following X-ray is taken. What should be done for this patient? A. DOT should be started while awaiting the results of a PPD test. B. He should be vaccinated or boosted with DTaP. C. A rapid Strep antigen test should be done to identify the cause of his pneumonia. D. He should be started on an oral course of oseltamivir. E. The large syncytia in his lungs should be removed surgically

DOT should be started while awaiting the results of a PPD test.

Which of the following is a difference between antigenic DRIFT and antigenic SHIFT in influenza viruses? A. DRIFT is somewhat predictable, SHIFT is not. B. SHIFT occurs because influenza virus RNA is not proofread. C. DRIFT can only occur in pigs or humans. D. SHIFT is responsible for the annual influenza outbreak. E. DRIFT alters the H and N designation of the influenza virus

DRIFT is somewhat predictable, SHIFT is not

This type of disease occurs at a low, but constant, frequency within a geographic area. A. endemic B. epidemic C. pandemic D. latent E. sequela

Endemic

Why is there not yet a vaccine directed against bacterial endotoxin? A. There is no market for one, so no one has tried hard enough to produce it. B. There are too many different varieties of endotoxin. C. Endotoxin is also a self-antigen in humans. D. Endotoxin does not produce a strong enough immune response. E. Endotoxin cannot be inactivated to produce a toxoid

Endotoxin cannot be inactivated to produce a toxoid

What is the function of the Staphylococcal protein A? A. IgA protease B. Fc receptor C. adhesin D. prevents phagosome / lysosome fusion E. activates complement C5a

Fc receptor

A common cause of both "pinkeye" and ear infections in children is . . . A. Human Herpesvirus 1 B. Streptococcus pyogenes (Group A strep) C. Haemophilus influenzae D. Bordetella pertussis E. Hantavirus

Haemophilus influenzae

Which of the following is a common cause of otitis media in children, as well as an important cause of more serious diseases such as meningitis and septicemia? A. Adenovirus B. Streptococcus pyogenes C. Haemophilus influenzae D. Rhinovirus E. Neisseria meningitidis

Haemophilus influenzae

An acute inflammatory respiratory disease associated with inhalation of aerosols from rodent urine, and especially prevalent in the southwest US is . . . A. Coccidioidomycosis B. Histoplasmosis C. Cryptosporidiosis D. Hantavirus E. Adenovirus

Hantavirus

The largest outbreak of this disease in history happened in Indianapolis in 1978 when the old Riverside Amusement Park was torn down. The buildings and rides had been unused since 1970, and had gathered a large volume of bird feces. Over 100,000 people were infected, 400 were hospitalized for respiratory distress and 15 died of a granulomatous disease. What was it? A. Influenza B. Histoplasmosis C. Coccidioidomycosis D. Hantavirus E. Tuberculosis

Histoplasmosis

How does interferon (IFN) function during a viral infection? A. Cells that produce IFN apoptose when infected by a virus. B. IFN is an oxidant that kills the virus by oxidizing viral capsid proteins. C. IFN is an inducer that turns on genes for antiviral proteins in neighboring cells. D. IFN is a cytotoxin that kills neighboring cells to prevent them from getting infected. E. IFN is a chemokine signal that induces antibody production by TC cells

IFN is an inducer that turns on genes for antiviral proteins in neighboring cells

How is IgA involved in the allergic response? A. It is needed in order to produce IgE. B. Its Fc part binds to Mast cells. C. It is released from Mast cells once they have recognized an allergen twice. D. It causes a greater inflammatory response than IgG does. E. IgA is not involved in the allergic response

IgA is not involved in the allergic reaction

Which antibody class can cross the placenta to protect the fetus? A. IgA B. IgD C. IgE D. IgG E. IgM

IgG

Which of the following describes a Type II hypersensitivity reaction? A. IgE causes cytokine release from T-lymphocytes. B. Excess IgG causes immune complexes to form. C. The first exposure to an antigen elicits as severe a response as subsequent exposures. D. IgG binds to a hapten on a cell, recruiting complement and NK cells. E. Effector TH cells activate TC cells against the body's own antigens

IgG binds to a hapten on a cell, recruiting complement and NK cells

During vaccine clinical trials, it was discovered that a vaccine antigen could bind to a receptor on the patient's epithelial cells. The vaccine development was cancelled. Why? A. The vaccine might prevent the disease agent from being recognized by phagocytes. B. The patient couldn't make antibodies to this sort of antigen. C. The vaccine antigen might activate a Type IV hypersensitivity reaction. D. IgG bound to the vaccine antigen might direct ADCC against the patient's own cells. E. The patient's TC cells might destroy the vaccine antigen, making it ineffective

IgG bound to the vaccine antigen might direct ADCC against the patient's own cells

The antibody type that is secreted by plasma cells one day after primary infection would most likely be: A. IgA B. IgD C. IgG D. IgM E. IgE

IgM

Which of the following antibody types is correctly matched with a feature of that antibody? A. IgM - only Ig produced in response to T-independent antigens B. IgG - first Ig produced in response to an infection C. IgD - FC part dimerizes to allow additional agglutination sites D. IgA - can cross placenta to protect developing fetus E. IgE - long half life allows long-term protection from circulating pathogens

IgM - only Ig produced in response to T-independent antigens

Class switching in B cells from _______ to _______ is responsible for the allergic response. A. IgG to IgM B. IgA to IgG C. IgM to IgE D. IgE to IgG E. MHC2 to MHC1

IgM to IgE

What is a virulence factor of the organism that causes the dry, itchy, contact-transmitted skin infection shown at the right? A. An A-B toxin B. An alpha-hemolysin C. A tissue-specific adhesin D. Keratinase E. Ability to prevent phagosome-lysosome fusion

Keratinase

Which of the following statements about live attenuated vaccines (LAV) and inactivated vaccines (INV) is correct? A. LAV contain the actual disease agent, which can induce a strong immune response. B. INV fail to induce B-cell responses, and so very low antibody titers are produced. C. LAV must always be given with adjuvants to maximize the humoral response. D. INV allow a patient to translate antigens from injected pathogen DNA. E. LAV can be either orally administered or injected, whereas INV must be injected.

LAV can be either orally administered or injected, whereas INV must be injected

The genes for which of the following would be LEAST likely to be located on a pathogenicity island? A. A Type III secretory system B. A bacterial siderophore C. A superantigen toxin D. An I/C factor E. LPS biosynthesis

LPS biosynthesis

Which complement-associated term is NOT correctly matched with its function? A. C3b - enhances phagocytosis B. Lectin - digests the C5 protein C. C3a - induces diapedesis D. C5b - recruits a lytic pore complex E. C3b - initiates one of the complement pathways

Lectin - digests the C5 protein

At the right is a photomicrograph showing the bacterium Listeria monocytogenes (Lm) infecting a human cell. What can you conclude about how this bacterium lives in its host? A. Lm must have an M-protein on its surface. B. Lm grows into long, filamentous cells once it is inside its host. C. After it passes through MALT tissue, Lm invades the epithelial basal membrane. D. Lm probably has an antiphagocytic capsule. E. Lm is able to use actin from its host to avoid humoral immunity.

Lm is able to use actin from its host to avoid humoral immunity

An example of an emerging or re-emerging disease is . . . A. Malaria in Africa B. Histoplasmosis in Indiana C. the annual Influenza epidemic D. Lyme Disease in the United States E. Polio in Europe

Lyme Disease in the United States

Which of the following can be diagnosed by a characteristic rash and is usually transmitted to humans by a biological vector? A. Lyme Disease B. Staphylococcal Scalded Skin Syndrome C. Diphtheria D. Sporotrichosis E. Papillomavirus

Lyme disease

Correctly order the following steps in the Shigella infection cycle. A. M cell entry → macrophage entry → epithelial cell entry → actin polymerization B. macrophage entry → actin polymerization → epithelial cell entry → bacteremia C. M cell entry → actin polymerization → epithelial cell entry → macrophage entry D. macrophage entry → bacteremia → toxin production → epithelial cell entry E. epithelial cell entry → actin polymerization → macrophage entry → bacteremia

M cell entry → macrophage entry → epithelial cell entry → actin polymerization

Which of the following correctly distinguishes MHC-I from MHC-II? A. MHC-I picks up an antigen as it passes through the phagloysosome. B. MHC-II displays only endogenous antigens. C. MHC-I can display self antigens on the surface of uninfected cells. D. MHC-II binds to TC cells, which destroy an infected cell. E. MHC-I is found only on professional phagocytes and B cells.

MHC-I can display self antigens on the surface of uninfected cells

We said that MHC2 is mainly for presentation of exogenous antigens, whereas MHC1 is mainly for presentation of endogenous antigens. Why is this so? A. MHC2 is on the outside of cells, MHC1 is on the inside. B. MHC2 binds to bacteria to allow macrophages to engulf the bacteria. C. MHC1 is found in the nucleus of infected cells, MHC2 in the cell membrane. D. MHC1 is involved in elimination of antibodies that recognize self antigens. E. MHC2 passes through the endocytic vesicle on its way to the cell surface

MHC2 passes through the endocytic vesicle on its way to the cell surface

Which of the following is associated with antigen presentation by dendritic cells , but not by epithelial cells? A. MHC1 picks up an external antigen circulating in the blood serum. B. MHC2 picks up an antigen from the phagocyte's cytoplasm. C. MHC1 picks up an antigen from the phagolysosome. D. MHC2 picks up an external antigen circulating in the blood serum. E. MHC2 picks up an antigen from the phagolysosome.

MHC2 picks up an antigen from the phagolysosome

Which of the following is associated with antigen presentation by phagocytes, but not by epithelial cells? A. MHC1 picks up an external antigen circulating in the blood serum. B. MHC2 picks up an antigen from the phagocyte's cytoplasm. C. MHC1 picks up an antigen from the phagolysosome. D. MHC2 picks up an external antigen circulating in the blood serum. E. MHC2 picks up an antigen from the phagolysosome

MHC2 picks up an antigen from the phagolysosome

After a visit from her 4-year-old granddaughter Sally, who had an ear infection, an elderly nursing home resident, Mary, developed pneumonia. What would have been the most effective way to prevent this infection? A. Use of drainage tubes to reduce the severity of Sally's ear infection B. Sally should have been treated with penicillin C. Mary should have been vaccinated with a 23-valent vaccine. D. Since this is transmitted by direct contact, Mary should not have hugged Sally. E. This is viral; nothing could have been done to prevent it

Mary should have been vaccinated with a 23-valent vaccine

Which of the following is a childhood disease that is potentially serious because it leaves a patient immunocompromised. It is now rare in the United States because of an effective vaccine. A. Zoster B. Group A Streptococcus C. Measles D. German Measles E. Chickenpox

Measles

A college student presented to a campus clinic with a low grade fever and a cough that had developed over the course of a few weeks. She said she felt tired. No lymph node swelling was apparent. A chest X-ray is shown at right. A sputum sample was stained, but no organisms were visible. The diagnosis would most likely be . . . A. Mycoplasmal pneumonia B. Streptoccal pneumonia C. Pertussis D. Histoplasmosis E. Tuberculosis

Mycoplasmal pneumonia

Which of the following statements about Human Papillomavirus (HPV) is correct? A. Most HPV infections can eventually cause cancer. B. HPV causes only some types of warts; other warts are caused by fungi. C. The Gardasil vaccine is effective against all varieties of HPV. D. Because it is protected by a tough envelope, HPV is difficult to destroy. E. Only a few HPV types can insert their DNA as a provirus into the host's chromosome

Only a few HPV types can insert their DNA as a provirus into the host's chromosome

Which of the following is transmitted more commonly as a community-acquired infection than as a nosocomial infection? A. Urinary tract infection due to E. coli B. Otitis media due to Haemophilus influenzae C. Scalded skin syndrome due to MRSA D. Catheter-associated septicemia due to Enterococcus E. Biofilm-associated Staphyloccus epidermidis infection

Otitis media due to Haemophilus influenzae

Rapidly spreading abscess and lymphatic swelling after a dog or cat bite would most likely be caused by . . . A. Bartonella B. Rubeola C. Rubella D. Pasteurella E. Varicella

Pasteurella

Which of the following organisms is NOT found more commonly in a nosocomial setting than as a community-acquired disease? A. Pseudomonas aeruginosa B. Pasteurella multocida C. Methicillin resistant Staph aureus D. urinary tract Escherichia coli E. Enterococcus faecalis (Group D Strep)

Pasteurella multocida

Your patient is an 8-year-old boy who has been bitten on the hand by a cat. He noticed a red area around the bite site a few hours after it happened, but wasn't concerned. A few hours after that, the red area had begun to spread along his arm, and it really hurt, so he told his mom, who brought him to you. You suspect that this is the same disease agent that causes respiratory distress diseases in livestock. What is it? A. Clostridium perfringens B. Bartonella henselae C. Pasteurella multocida D. Clostridium tetani E. It is likely an A-B intoxication, not an organism

Pasteurella multocida

Elderly patients on long-term oral penicillin therapy often acquire the infectious disease pseudomembranous colitis, caused by Clostridium difficile. How does penicillin use affect the course of this disease? A. Penicillin kills native flora, allowing C. difficile to colonize. B. Penicillin creates a "super" strain of C. difficile that is more infectious. C. Penicillin triggers a hypersensitivity response in the patient. D. Penicillin reduces the severity of the C. difficile infection. E. Penicillin has no effect on the course of this disease

Penicillin kills native flora, allowing C. difficile to colonize

An important disease of childhood that has increasing prevalence in the US due to insufficient immunity in adults A. Adenovirus B. Rhinovirus C. Influenza D. Pertussis E. Respiratory syncytial virus

Pertussis

Which of the following may appear in many places in the body, all of which usually involve biofilms, often with a greenish pigment, forming on inadequately protected tissues? A. Lyme Disease B. Gangrene C. Staphylococcus D. Impetigo E. Pseudomonas

Pseudomonas

Which of the following could best be described as an opportunistic pathogen with high levels of R-plasmid-mediated antibiotic resistance whose biofilms colonize patients with a weakened innate immune system? A. Streptococcus pyogenes B. Clostridium tetani C. Borrelia burgdorferi D. Pseudomonas aeruginosa E. Human Papillomaviru

Pseudomonas aeruginosa

In order to prevent nosocomial infections, it is important to know which infections are the most common so we can focus our efforts where they will do the most good. Which of the following is (by far) the most common nosocomial infection in the United States? A. E. coli urinary tract infections from indwelling catheters B. Pseudomonas respiratory tract infections from contaminated air ducts C. Staphylococcal infections of wounds and surgical incisions D. Streptococcal bacteremia from contaminated I.V. lines E. Fungal skin infections from contact with contaminated surfaces

Pseudomonas respiratory tract infections from contaminated air ducts

A 3-week-old infant was brought to the emergency room in respiratory distress. Inflamed bronchioles were found when she was examined, and the following cell structures were seen in a microscope slide prepared from bronchiole epithelial cells. What is the most likely cause of her illness? A. Coccidioidomycosis B. Hantavirus pulmonary syndrome C. Respiratory Syncytial Virus D. Diphtheria E. Whooping Cough

Respiratory Syncytial Virus

An 18-month-old child developed bronchiolitis, and his mother took him to a clinic. The Doctor saw that he was in obvious respiratory distress, but told the mother that there was nothing that could be done but to keep his airway moist and hope that he recovered. This was the second time the infant had been affected by the same illness. It is most likely . . . A. Hantavirus B. Respiratory Syncytial Virus C. Haemophilus influenzae D. Pertussis E. Cryptosporidium

Respiratory Syncytial Virus

The most common cause of lower respiratory tract infections in young children causes irritation to the bronchiole lining, with the characteristically damaged cellular appearance seen at right. What is this disease? A. Respiratory Syncytial Virus B. Influenza A C. Streptococcal pneumonia D. Mycoplasmal tuberculosis E. Coccidioidomycosis

Respiratory Syncytial Virus

The most common cause of viral pneumonia in young children is . . . A. Rubeola B. Hantavirus C. Influenza D. Respiratory Syncytial Virus E. Rubella

Respiratory Syncytial Virus

Which of the following disease organisms is correctly matched with the main symptoms of the disease that it causes? A. Pasteurella multocida - a circular or annular ("bull's eye") rash B. Rubella - congenital syndrome with blindness, deafness and mental retardation C. Varicella-Zoster Virus - cough, conjunctivitis and coryza (runny nose) D. Bartonella henselae - spasms and rigid paralysis of all skeletal muscles E. Hantavirus - painful blisters on the head or body of adults who have had chicken-pox

Rubella - congenital syndrome with blindness, deafness and mental retardation

All of the following diseases EXCEPT ___________ are more serious in the fetus because of the fetus's lack of cellular immunity. A. Rubeola B. Rubella C. Varicella D. Toxoplasma E. Herpes

Rubeola

Occasionally fatal virus transmitted by respiratory droplets and identified by a characteristic rash, which includes spots on the oral mucosa. The virus can multiply in T cells, and most of the fatality is due to secondary infections in the T-cell-suppressed host A. Bartonella B. Rubeola C. Rubella D. Pasteurella E. Varicella

Rubeola

This organism is associated with a frequently fatal respiratory syndrome with capillary leakage leading to pulmonary edema. It is spread primarily in the southwest United States by inhalation of dust aerosols contaminated with rodent urine or feces. A. Adenovirus B. Sin Nombre hantavirus C. Tinea corporis D. Histoplasma capsulatum E. Bartonella henselae

Sin Nombre hantavirus

One form of the organism that causes the symptoms shown in the picture enters the body on the point of a thorn. It then changes to another form and progressively spreads along a lymphatic vessel. What is the disease? A. Thrush B. Coccidioidomycosis C. Trichomonas D. Sporotrichosis E. Papillomavirus

Sporotrichosis

A woman who had shaved only her lower legs all winter decided to shave her thighs to prepare for summer. A few days later she developed small red bumps all over her thighs. What is likely the cause? A. Streptococcal erysipelas B. Recurrence of Zoster from a previous Chickenpox infection C. A shaving cut has become infected with necrotizing fasciitis. D. Streptococcal or Staphylococcal impetigo E. Staphylococcal folliculitis

Staphylococcal folliculitis

Several infants in the neonatal ward of a hospital show symptoms including red, inflamed skin with peeling of the epidermal layer. They are likely infected with . . . A. Pseudomonas aeruginosa B. Streptococcus pyogenes C. Sporothrix schenckii D. Staphylococcus aureus E. Rubeola virus

Staphylococcus aureus

Infections with which of these organisms are most associated with biofilm formation? A. Staphylococcus aureus and Streptococcus pneumoniae B. Streptococcus pneumoniae and Clostridium perfringens C. Staphylococcus epidermidis and Pseudomonas aeruginosa D. Pseudomonas aeruginosa and Streptococcus pneumoniae E. Streptococcus pyogenes and Sporothrix schenckii

Staphylococcus epidermidis and Pseudomonas aeruginosa

A 63-year-old man had a hip implant a year ago. He has recently experienced pain in the hip, and suddenly developed a high fever and septicemia. The implant was removed, and its surface was scraped with an inoculating loop and plated on Blood agar. The growth is shown at the right. The organism most likely responsible for the infection is _____________ and its chief virulence factor is _____________. A. Staphylococcus aureus; beta hemolysis B. Viridans group Streptococcus; coagulase C. Group A Streptococcus; toxic shock syndrome toxin D. Staphylococcus epidermidis; ability to form biofilms E. Clostridium perfringens; an AB exotoxin

Staphylococcus epidermidis; ability to form biofilms

A college student who was a cigarette smoker presented to a campus clinic with a high fever and a hacking cough that had developed over the course of a few days. Extreme shortness of breath was apparent. A sputum sample showed PMNLs and Gram positive cocci. The diagnosis would most likely be . . . A. Mycoplasmal pneumonia B. Streptoccal pneumonia C. Pertussis D. Histoplasmosis E. Tuberculosis

Streptococcal pneumonia

Immune tolerance in T cells involves positive selection. What does that mean? A. T cells are only released if their MHC1 recognizes antibodies in the thymus. B. T cells are only released if their TCR recognizes thymus cells' MHC1. C. T cells are only released if their MHC2 recognizes macrophages in the thymus. D. T cells are not released if their antibodies recognize antigen in the thymus. E. T cells are not released if their TCR recognize thymus cells' MHC2

T cells are only released if their TCR recognizes thymus cells' MHC1

What is a major difference between TH and TC cells? A. TC can bind to almost any infected cell; TH only bind infected antigen presenting cells B. TC mostly secrete cytokines; TH secrete histamine C. Effector TH cells induce apoptosis; effector TC cells induce humoral immunity D. TH cells produce B7, but TC cells produce interleukin-2 E. TH cells undergo clonal deletion; TC undergo clonal expansion

TC can bind to almost any infected cell; TH only bind infected antigen presenting cells

Humans with the mutation IL<tm1hor> fail to produce interleukin-2. Which of the following would you expect to be a direct result of this mutation? A. B cell function would be impaired. B. TC cells could only be activated by binding to infected APCs. C. T cells that recognize self-antigens would remain in the body. D. T cells would be unable to produce antibodies unless stimulated directly by B cells. E. TH cells could never mature and differentiate into TC cells.

TC cells could only be activated by binding to infected APCs

What is the most correct statement about the selection process T cells must undergo before they are released to the blood? A. V-D-J combinations do not occur if they would produce TCRs against self antigens. B. TCRs must recognize self MHCs, but must not recognize antigens on the MHCs. C. Recognizing any circulating antigen causes T cells to become anergic. D. Recognizing any antigens in bone marrow causes T cells to apoptose. E. T cells fail to multiply in the thymus unless they recognize a self antigen on an MHC1

TCRs must recognize self MHCs, but must not recognize antigens on the MHCs

What is the difference between TH and TC cells? A. TH cells have MHC1 on their surface, TC have MHC2 B. TH cells recognize antigens, TC cells recognize antibodies. C. TH cells secrete cytokines, TC cells secrete perforin. D. TC cells need two signals for them to become "effector" T cells; TH only need one. E. TC cells bind only to APCs, TH can bind to any other cell.

TH cells secrete cytokines, TC cells secrete perforin

What is the role of T cells in the antibody response? A. TH cells initiate the V-D-J joining process. B. TH cells engulf antigens for presentation to B cells. C. TH cells stimulate B cell division and differentiation. D. TC cells secrete one of the types of antibodies. E. T cells play no role in the antibody response.

TH cells stimulate B cell division and differentiation

Which of the following is most responsible for the symptoms associated with poison ivy? A. Autoimmune antibodies B. TC cells that recognize both poison ivy and your own antigens C. An immunodeficiency disease you have before you contact the poison ivy D. TH cells that activate macrophages in the skin E. Bacteria that live on the poison ivy plants

TH cells that activate macrophages in the skin

The following table tries to define the differences between TH and TC cells, but it gets a lot of things wrong. Which line in the table is correct? TH cells TC cells A. T cell binds to . . . APCs only any cell type B. T cell stimulates . . . IgG production IgA production C. T cell surface protein . . . CD28 B7 (CD80) D. T cell presents antigens on . . . MHC-II MHC-I E. T cell recognizes . . . foreign antigens self antigens

Tcell binds to...APCs only, any cell trype

A TB test involves a Type IV hypersensitivity reaction. If you have been sensitized to TB before you take such a test . . . A. anti-TB antibodies circulating in your body will cause the test to be positive B. Mast cells in your body will be coated with antibodies against TB C. your macrophages will have TB living inside them, and this will cause a positive test D. Tmemory cells in your body that recognize TB will cause the test to be positive E. it won't matter, since sensitization is not required for Type IV hypersensitivity

Tmemory cells in your body that recognize TB will cause the test to be positive

Which T cell type is correctly matched with a function? A. TH cells - Have surface receptors that recognize a pathogen B. Tregulatory cells - Decrease the intensity of the cellular immune response C. TC cells - Secrete interleukin-2 to allow other T cells to divide D. Tmemory cells - Produce IgG immediately upon infection E. TH cells - Bind to the FC part of antibodies to cause ADCC

Tregulatory cells - Decrease the intensity of the cellular immune response

An example of an emerging or re-emerging disease is . . . A. Cholera in India B. Coccidioidomycosis in New Mexico C. the annual Influenza epidemic D. Tuberculosis in Europe E. Gonorrhea in the United States

Tuberculosis in Europe

In the disease lupus, your B cells recognize your own DNA, making antibodies that form antigen-antibody complexes with DNA. This is an example of which type of hypersensitivity? A. Type I B. Type II C. Type III D. Type IV E. It's not a hypersensitivity reaction, but it causes DNA damage

Type III

In the disease rheumatoid arthritis, a person makes antibodies against her own collagen. Antibody-antigen complexes form in the capillaries of joints, causing inflammation leading to arthritis. This is an example of . . . A. Type I hypersensitivity B. Type II hypersensitivity C. Type III hypersensitivity D. Type IV hypersensitivity E. an immunodeficiency disease

Type III hypersensitivity

A person has a Staphylococcal skin infection. Another person acquires the same infection by sleeping on the same sheet as the first person. The sheet in this scenario serves as . . . A. A vector B. A reservoir C. A fomite D. A portal of entry E. A host

a fomite

One day, an ICP for a large hospital in Chicago recorded 43 cases of a particular infection in the hospital, including visitors as well as patients. This infection is not endemic to the area. The following day there were 12 new cases. No new cases were recorded after that. The most likely source of this outbreak was . . . A. a visitor to the hospital who did not wash his hands B. a kitchen that prepared food for the patients and for the cafeteria C. a new patient admitted to the hospital with this infection D. a nurse or doctor who is an asymptomatic carrier for the disease E. contaminated tap water from the city's water treatment plant

a kitchen that prepared food for the patients and for the cafeteria

Antigenic drift in influenza virus is . . . A. a major reason for worldwide pandemics B. a reassortment of H and N proteins between two viruses that infect the same host C. the mechanism whereby influenza spreads from person to person around the world D. possible because the flu virus has a segmented genome E. a minor change in the virus spike proteins caused by mutation of the RNA

a minor change in the virus spike proteins caused by mutation of the RNA

If you travel to the former Soviet Union and return with a case of Diphtheria, the Doctor will likely give you a shot of Diphtheria Immune Globulin. What is that? A. An antibody against a bacterial surface protein B. An injected antibiotic C. A toxoid vaccine D. An active form of the diphtheria antigen E. A passive antitoxin

a passive antitoxin

In a recent mumps outbreak the case incidence rate over time is recorded as shown (x-axis is in weeks). This is an example of . . . A. A common-source epidemic B. Fecal-oral transmission C. Delayed hypersensitivity D. A person-to-person epidemic E. An endemic disease

a person-to-person epidemic

The bacterium Listeria monocytogenes can live in phagocytes by breaking down the phagolysosome membrane. Which of the following toxins could help L. monocytogenes do this? A. A hyaluronidase B. A superantigen C. Endotoxin D. An AB toxin E. A phospholipase

a phospholipase

A flight lands at O'Hare with a passenger who has a respiratory disease. A week later, there are 155 cases of the disease in Chicago. By the second week, there are 6,000 cases and an epidemiologist is consulted. He would most likely consider this to be . . . A. a propagated epidemic of an airborne disease B. a person-to-person epidemic with multiple index cases C. a common-source epidemic D. an example of an emerging disease E. a disease transmitted by respiratory droplets

a propagated epidemic of an airborne disease

How is interferon produced during an infection? A. It is produced by induction of viral genes in the phagolysosome. B. Bacteria produce it in response to the phagocyte oxidative burst. C. A second-messenger pathway induces it when viral RNA binds to an RLR receptor. D. It is produced when MHC-I phosphorylates a response regulator in infected cells. E. TH cells produce it when they bind to MHC-II that is displaying viral antigens

a second-messenger pathway induces it when viral RNA binds to an RLR receptor

What is an adjuvant? A. A toxin that has been inactivated by formaldehyde crosslinking B. A type of vaccine in which DNA is injected into your cells C. A substance in some vaccines that helps to elicit a cellular immune response D. A substance that must be added to live vaccines to attenuate them E. A young lymphoid stem cell before it becomes either a TH or a TC cell

a substance in some vaccines that helps to elicit a cellular immune response

What is the vaccine against Streptococcal pneumonia called? A. A live attenuated vaccine B. A toxoid vaccine C. A subunit conjugate vaccine D. An oral recombinant vaccine E. An artificial passive vaccine

a subunit conjugate vaccine

What is a leukocidin? A. a type of leukocyte B. a type of stem cell found in the blood C. a toxin produced by bacteria to kill macrophages D. a toxin produced by macrophages to kill bacteria E. a cytokine that induces apoptosis in infected cells

a toxin produced by bacteria to kill macrophages

Which of the following is the best example of balanced pathogenicity? A. an organism that causes acute respiratory disease that is rapidly fatal B. a foodborne organism that causes septicemia C. both bacteria and viruses can cause pneumonia D. a virus that causes mild diarrhea and is spread fecal-oral with an ID50 of 20 organisms E. a bacterium that causes an equal number of respiratory and skin diseases

a virus that causes mild diarrheas and is spread fecal-oral with an ID50 of 20 organisms

The infant shown in this picture has an itchy red rash that is making him fussy and irritated. The organism that is most likely causing this rash is . . . A. a dimorphic fungus like Sporothrix B. a yeast like Candida C. a dermatophyte like ringworm D. the Varicella virus E. Bartonella henselae

a yeast like Candida

Which of the following requires the strictest isolation between an infected patient and a healthcare worker? A. Standard precautions B. Airborne precautions C. Droplet precautions D. Contact precautions E. Blood and body fluid precautions

airborne precautions

Which of the following could be considered to be a phagocyte? A. TC cell B. TH cell C. lymphocyte D. antigen presenting cell E. mast cell

antigen presenting cell

Which parts of the antibody shown at the right undergo somatic recombination? A. all 4 parts labeled 'A' B. both parts labeled 'B' C. all 4 parts labeled 'C' D. all 4 parts labeled 'D' E. The whole antibody undergoes somatic recombination with other antibodies

all 4 parts labeled 'A'

Several organisms (Staphylococcus, Pseudomonas, Enterococcus) cause disease in catheterized patients much more frequently than in non-catheterized patients. What is the most important virulence factor for these organisms? A. Antiphagocytic capsule B. Exotoxin production C. Respiratory transmission D. Intracellular spread via actin polymerization E. Ability to form biofilms on the catheter

ability to form biofilms on the catheter

Antibodies can "fight" bacterial infections in all of the following ways EXCEPT . . . A. stop bacterial motility B. attract NK cells C. direct a pore complex to form in the bacterial membrane D. activate TH cells to secrete cytokines E. make bacteria easier for phagocytes to recognize

activate TH cells to secrete cytonkines

Which of the following is NOT part of an allergic response? A. A B-cell binds the allergen and is activated by a TH cell. B. The B-cell turns into a plasma cell. C. Circulating antibodies bind to mast cells. D. Activated B cells bind allergen a second time. E. Allergen binds simultaneously to at least two antibodies on mast cells

activated B cells bind allergen a second time

The antibody shown below can help your body "fight" against pathogens in all of the following ways EXCEPT . . . A. neutralization of toxins B. immobilization of bacteria C. activation of ADCC D. agglutination of viruses E. interfering with bacterial adhesion to surfaces

activation of ADCC

What is primarily responsible for the current trend in Pertussis incidence in the USA? A. A better vaccine that has fewer side effects B. Spread of resistant forms of the virus that causes Pertussis C. Adult immunity decreasing below the herd immunity threshold D. Increased tourism to areas of the world where Pertussis is endemic E. Requirement for college students to be vaccinated if they live in dormitories

adult immunity decreasing below the herd immunity threshold

Bob has had tuberculosis. If a TB skin test is performed on Bob, what will happen? A. He will show a red rash within a few minutes at the site where the test was performed. B. He will show a red rash over his whole body within a few hours. C. After a few days he will show a red rash at the site where the TB antigen was injected. D. He will form immune complexes to the injected antigen. E. Nothing, unless his TB is active at the time of the test

after a few days he will show a red rash at the site where the TB antigen was injected

In general, antibodies can defend against pathogens in any of six ways. However, IgM and IgA can only use a few of these methods. Which of the following could be used by IgM or IgA to inactivate pathogens? A. Agglutination B. Opsonization C. Complement activation D. NK cell activation E. TH cell activation

agglutination

The major symptoms in a case of diphtheria are caused by . . . A. the body's immune response to the infection B. growth of the bacterium across the tracheal opening C. an AB toxin that interferes with protein synthesis D. viral damage to the mucociliary escalator E. a phospholipase toxin that lyses blood cells and steals their iron

an AB toxin that interferes with protein synthesis

Which of the following is an example of vertical transmission of a disease? A. Fourteen people on an airplane become ill when a sick passenger is allowed to fly. B. A nurse with MRSA passes it to several babies in the neonatal ward of a hospital. C. Second-graders get Norovirus when a lunch worker forgets to wash her hands. D. Three children in a family all get a cold from their Mom. E. An HIV-positive baby is born to a mother with AIDS.

an HIV-positive baby is born to a mother with AIDs

The vaccine currently used to provide protection against whooping cough is . . . A. A passive vaccine B. A polysaccharide conjugate vaccine C. An acellular subunit vaccine D. A live attenuated vaccine E. A whole-cell inactivated vaccine

an acellular subunit vaccine

A six-year-old girl has been brought to Urgent Care. She had had a fever and runny nose and then had begun coughing. The coughs had become so severe that she began vomiting, and her mother was afraid she couldn't breathe. Which of the following vaccines could have prevented this disease? A. an inactivated whole cell vaccine against Streptococcus B. an attenuated live virus vaccine given as a nasal spray C. a toxoid vaccine given orally D. an acellular subunit vaccine conjugated to another vaccine E. none - there is no vaccine against this disease

an acellular subunit vaccine conjugated to another vaccine

We mentioned in class that a lot of money is being spent to develop a protein-conjugate vaccine that recognizes 13 of the 80 different varieties of Streptococcus pneumoniae. But there is already a polysaccharide vaccine that recognizes 23 of the 80 varieties. What is the advantage of the new vaccine? A. An epitope of the conjugated protein can be recognized by TH cells. B. The protein binds to more B cell receptors than the polysaccharide does. C. The protein is a better PAMP than the polysaccharide is. D. NK cells recognize the protein better than they do the polysaccharide. E. There is actually more immune tolerance if fewer varieties of S. pneumoniae are recognized.

an epitope of the conjugated protein can be recognized by TH cells

Who or what is a fomite? A. A dried sputum droplet that can become aerosolized B. An exotoxin secreted by Streptococcus C. The first person to become ill during an epidemic D. An inanimate object that spreads infection via indirect contact E. An air-filled vacuole within macrophages where bacteria are stored before digestion

an inanimate object that spreads infection via indirect contact

The bacterium Legionella pneumophila causes an acute bacterial pneumonia with an incubation period of 3 days (not one we discussed in class). It can often be diagnosed only by serological tests. A patient with pneumonia arrives at a Dr's office for various tests, including a titer for anti-Legionella IgG. The titer is 1:32. No diagnosis is made, but the patient eventually recovers. The Dr still wants to know what caused the illness, though, so she runs another IgG titer. What test would be best for the IgG titers mentioned in the question above? A. a precipitin test B. indirect immunoelectrophoresis C. a direct ELISA test D. an indirect ELISA test E. a direct fluorescent antibody test

an indirect ELISA test

What is a major difference between an infection and an intoxication? A. An infection is best treated by inducing active immunity in an ill patient. B. An intoxication is best treated by passive immunization. C. An intoxication usually results in disorientation and neural paralysis. D. An infection is more acute than an intoxication. E. The consequences of an intoxication are not as serious as those of an infection

an intoxication is best treated by passive immunization

Granuloma formation involves all of the following EXCEPT . . . A. antibodies B. cytokines C. macrophages D. bacteria E. T cells

antibodies

The most serious symptoms of Lyme disease are caused by . . . A. antibodies made against the bacterium that causes the disease B. a bacterial AB toxin that interferes with cAMP balance C. production of gas in tissues, which cuts off the blood supply and causes ischemia D. granuloma formation in the lung and other tissues E. a hyaluronidase toxin that inhibits host cell protein synthesis

antibodies made against the bacterium that causes the disease

It is important to treat Strep throat aggressively as soon as it is noticed. Why? A. It could develop into pneumonia if allowed to persist too long. B. It could invade the throat tissues and cause necrosis. C. Antibodies produced against it could damage heart valves and kidneys. D. The longer it persists, the more likely you are to get it again. E. The virus that causes it can also cause meningitis.

antibodies produced against it could damage the heart valves and kidneys

Which of the following is a mechanism that bacteria use to avoid the host's immune response? A. Bacteria digest IgG B. Bacteria secrete iron C. Bacteria induce autoimmmune reactions D. Bacteria cover themselves with antibodies E. Bacteria cover themselves with C3b

bacteria cover themselves with antibodies

Throughout the 20th century, we have had to modify Koch's postulates as we learned more about host-pathogen interactions. All of the following observations have required modification of Koch's postulates, EXCEPT . . . A. Occasionally a disease does not have an identifiable cause. B. Bacteria eventually evolve less virulence as hosts evolve more resistance. C. Most bacteria cannot be cultured in laboratory media. D. Some infections are caused by several bacteria growing together in a biofilm. E. Some bacteria that cause human disease do not cause the same disease in animals

bacteria eventually evolve less virulence as hosts evolve more resistance

Which of the following is true of infections that are characterized by granuloma formation? A. Bacteria that cause these infections lack PAMPs on their surface. B. APCs associated with these infections fail to engulf bacterial pathogens. C. Bacteria that cause these infections produce many leukocidins and oxidants. D. Bacteria prevent lysosomes in infected macrophages from fusing with phagosomes. E. Infected macrophages fail to produce MHC-II, and so are not recognized by IgG

bacteria prevent lysosomes in infected macrophages from fusing with phagosomes

During pre-clinical testing, a certain pharmaceutical drug is found to stimulate the production of interleukin-2 (IL-2) in human cell cultures. This finding is . . . A. good, because IL-2 allows V-D-J joining so that antibodies can be produced. B. bad, because IL-2 is involved in the Type II hypersensitivity response. C. good, because IL-2 is an important anti-viral defense. D. bad, because producing IL-2 without an infection will overstimulate TC cells. E. good, because IL-2 can act like a hapten to induce cellular immunity

bad, because producing IL-2 without an infection will overstimulate TC cells

A chronic disease with a seasonal incidence peak in the summer would most likely . . . A. have a prevalence peak in the winter B. be transmitted via respiratory droplets C. be called an "emerging disease" every summer D. be vector-borne, like Lyme Disease E. have incidence greater than its prevalence by the end of the summer

be vector-borne, like Lyme Disease

Inheritance of the immunodeficiency disease agammaglobulinemia results in the failure to produce B cells. A patient with this disease would . . . A. Be very susceptible to intracellular pathogens like Shigella B. Never develop an acquired immune response C. Be incapable of developing any type of hypersensitivity reaction D. Be very susceptible to bacterial toxins E. Be incapable of activating the complement system

be very susceptible to bacterial toxins

Which of the following isolation procedures commonly used in hospitals is the least rigorous (i.e. requires the minimum use of additional protective barriers between patient and healthcare worker)? A. Body substance precautions B. Droplet precautions C. Airborne precautions D. Contact precautions E. Sterile precautions

body substance precautions

What do the structures in these two pictures have in common? A. They are both parts of the same bacterial cell. B. Both involve rearrangements of the host's cytoskeletal proteins. C. Both occur when a bacterial capsule is produced within a host cell. D. Both are immune responses to the presence of a bacterium. E. Both are ways that Shigella can enter macrophages

both involve rearrangements of the host's cytoskeletal proteins

What do TH and TC cells have in common? A. They can both engulf bacterial PMPs. B. They can both bind to MHC1 to stimulate B cells. C. Both can secrete perforin to induce apoptosis in infected cells. D. Both produce the signaling molecule B7. E. Both need an infection-specific "second signal" to avoid becoming anergic

both need an infection-specific "second signal" to avoid becoming anergic

What do precipitin and agglutination tests have in common? A. Both require that the antibodies have at least two antigen binding sites. B. A positive test is indicated by a white line between the antigen and antibody spots. C. Both require diffusion of antigen and antibody through a gel. D. Both involve labeled secondary antibodies. E. Both involve electrophoretic separation of the antigens

both require that the antibodies have at least two antigen binding sites

Many of the organisms that cause eye infections also cause ear infections and sinusitis. There are important exceptions, though. Organisms that cause eye infections but NOT ear ot sinus infections are probably acquired . . . A. via infected aerosols B. from showering or bathing in infected water C. by contact with infected respiratory droplets D. during epidemics in the community E. by direct or indirect contact with an infected source

by direct or indirect contact with an infected source

A stroke victim is admitted to a hospital's intensive care unit and placed on a respiratory ventilator. She develops pneumonia characterized by green sputum and a high fever. How did she most likely acquire this infection? A. By contact with the patient in the adjoining bed B. From the unwashed hands of a health care worker C. By formation of a biofilm on the ventilator tube D. From bacteremia caused by a urinary catheter E. She should have been on airborne precautions because of the stroke

by formation of a biofilm on the ventilator tube

How can an antibody's titer be determined most easily? A. by observing its reaction with an antigen B. by diluting it and plating it on a Kirby-Bauer plate C. by filtering it and adding a dye to the filter D. by isolating it from blood and examining it by electron microscopy E. by adding it to a bacterial culture and watching the bacteria lyse

by observing its reaction with an antigen

How does a phospholipase toxin kill cells? A. By preventing phospholipid synthesis B. By removing the head groups, thus destroying the amphipathic nature of the phospholipid C. By adding ADP + ribose to stop protein synthesis D. By preventing uptake via membrane fusion E. By digesting fatty acids, causing the lipids to not pack together as well

by removing the head groups, thus destroying the amphipathic nature of the phospholipid

How does your body respond to the presence of a superantigen toxin? A. By secreting lots of cytokines and mounting a massive inflammatory response B. By producing many more antibodies than normal, causing hypersensitivity reactions C. By recruiting the complement cascade to kill host cells as well as bacteria D. By engulfing the toxin and disseminating it via the blood to the tissues E. By destroying connective tissue, resulting in necrosis.

by secreting lots of cytokines and mounting a massive inflammatory response

How does interferon prevent viral replication? A. By preventing the formation of dsRNA B. By signaling cells to be ready to apoptose if a virus invades C. By killing host cells after the virus has been assembled but before budding D. By preventing the virus from binding to host cell receptors E. By preventing the processing of viral polyproteins

by signaling cells to be ready to apoptose if a virus invades

Serious type IV hypersensitivity reactions are usually treated . . . A. with anti-histamines B. with immune globulin injections C. by suppressing T cell response D. by using one type of antibody to competitively inhibit another E. by using cytokines to enhance the innate immune response

by suppressing T cell response

A 30-year-old patient presents to a clinic with heart arrhythmia and joint pain. A few weeks previously, while camping in the Maine woods, he had noticed an unusual rash "like a bulls' eye" that had gone away. How had he likely become infected? A. by eating contaminated food B. by the bite of a small tick C. by drinking water contaminated with rodent urine D. through skin puncture or abrasion E. by inhaling fungal spores in an aerosol of dry leaves

by the bite of a small tick

The test shown below . . . A. is an example of an agglutination test B. can determine which bacterial surface molecules to use in a subunit vaccine C. is a typical Western Blot D. could represent a negative result (a patient who is not infected with a particular bacterium) E. is most likely used to compare a pre-immune serum to a convalescent serum

can determine which bacterial surface molecules to use in a subunit vaccine

Which of the following cell types is LEAST likely to be killed by NK cells? A. Cells in your body that are infected by viruses B. Cells in your body that are overproducing MHC-I C. Cells in your body that have a hapten stuck to their surface D. Bacterial cells that have invaded your body E. Cells in your body that have been transformed to become tumor cells

cells in your body that are overproducing MHC-1

The upper part of the healthy respiratory tract has a native flora, but the lower part is sterile. What is mainly responsible for this? A. the acidic pH at the back of the throat B. ciliated epithelial cells that line the upper respiratory tract C. the membranous folds of the vocal cords D. IgA produced in the upper respiratory tract E. rapid exhalation that expels organisms from the lungs

ciliated epithelial cells that line the upper respiratory tract

The process that ensures no circulating B cells recognize self antigens is called: A. class switching B. clonal selection C. somatic recombination D. clonal deletion E. conjugation

clonal deletion

Some bacteria live and divide inside the phagolysosome. What will be a symptom associated with a chronic infection of a patient with such a bacterium? A. Clumps of infected macrophages and TH cells will be seen in the patient's tissues. B. There will be a lower-than-normal number of phagocytes in the patient's body. C. The patient will suffer from constant high fever. D. The patient will not produce antibodies against the bacterium. E. The patient will suffer hypovolemia and DIC

clumps of infected macrophages and TH cells will be seen in the patient's tissues

What are "pathogenicity islands"? A. Clusters of virulence factor genes that can be transmitted by horizontal gene transfer B. Groups of pathogenic bacteria bound to M cells C. Patches of membrane receptors to which pathogenic bacteria bind D. Plasmids on which multiple antibiotic resistance genes are located E. Endemic disease foci with high prevalence of a particular disease

clusters of virulence factor genes that can be transmitted by horizontal gene transfer

What are "pathogenicity islands"? A. Patches of membrane receptors to which pathogenic bacteria bind B. Plasmids on which multiple antibiotic resistance genes are located C. Clusters of virulence factor genes that can be transmitted by horizontal gene transfer D. Groups of pathogenic bacteria bound to M cells E. Endemic disease foci with high prevalence of a particular disease

clusters of virulence factor genes that can be transmitted by horizontal gene transfer

This organism has two spores in its life cycle - endospores, released in the body from spherules, and the distinctive spores at right, which are inhaled from the environment. A. Histoplasmosis B. Hantavirus C. Coccidioidomycosis D. Trichonympha E. Cryptococcosis

coccidioidomycosis

How does a Type III hypersensitivity reaction damage your tissues? A. Misdirected TC and NK cells kill your own cells. B. Macrophages recruited to the site of an infection start to digest your own cells. C. Complement activation in capillaries recruits PMNs, which secrete perforins. D. Infectious organisms survive within your macrophages, causing inflammation. E. TH cells direct the release of mediators that cause fluid leakage from capillaries

complement activation in capillaries recruits PMNs, which secrete perforins

Adenovirus is commonly cited as a way to deliver genes or drugs into human cells, because it naturally infects our cells. Which 3 symptoms does this virus usually cause? A. Conjunctivitis, cold and diarrhea B. Sore throat, pyelonephritis and rheumatic fever C. Pneumonia, diarrhea and vomiting D. Cold, ear infection and meningitis E. Diarrhea, pneumonia and warts

conjunctivitis, cold, and diarrhea

Which of the following symptoms would be considered unusual in a case of Strep throat? A. swollen lymph nodes B. tonsillitis C. red petechial patches in the throat D. cough E. pus production in the back of the throat

cough

How are TH cells involved in the humoral immune response? A. TH cells produce antibodies B. Macrophages produce antibodies in response to TH cells C. Antibodies are modified by TH cells so that they are active D. TH cells kill infected cells by releasing perforin and inducing apoptosis E. Cytokines from TH cells cause B cells to turn into antibody-producing plasma cells

cytokines from TH cells cause B cells to turn into antibody-producing plasma cells

A 9-year-old boy has a cough, fever, and runny nose. He also has a fine, macular rash and small white spots in his mouth. What is the danger from this disease? A. Damage to his T-cells may lead to serious secondary infections. B. The rash might spread to internal organs, causing organ failure and DIC. C. The causative virus is latent, and may recur later as a more serious disease. D. Antibodies he makes against the causative bacterium might be responsible for autoimmune sequelae. E. If his mother is pregnant, his unborn sibling may develop congenital abnormalities.

damage to his T-cells may lead to serious secondary infections

The protein called B7 is produced by ________ to ensure that an immune response is only produced in response to the presence of a pathogen. A. TH cells B. B cells C. dendritic cells D. bacteria E. all cells with nuclei

dendritic cells

What molecule is typically recognized by the immune system to signal that a cell has been infected by a virus? A. Complement protein C3b B. Viral capsomeres C. Viral envelope lipids D. Viral spike proteins E. Double stranded RNA

double stranded RNA

Which of the following is an important difference between droplet transmission and aerosol transmission of a disease? A. Aerosol transmission is limited to hosts who come into direct contact. B. Only aerosol transmission results in an epidemic disease. C. Droplet transmission almost always occurs via fomites. D. Droplet transmission rarely results in a common-source outbreak. E. Aerosol transmission is almost exclusively for digestive system diseases

droplet transmission rarely results in a common-source outbreak

How do bacteria with Type III secretion systems (T3SS) typically enter host cells? A. The T3SS pokes a hole in the host cell membrane through which the bacteria enter. B. The T3SS kills the host cell, making it easier for the bacteria to enter. C. The T3SS enhances binding to macrophages, which engulf the bacteria. D. Effectors secreted through the T3SS cause the host cell to enfold the bacteria in membrane ruffles. E. They don't. T3SS are for secreting exotoxins, not for entering host cells

effectors secreted through the T3SS cause the host cell to enfold the bacteria in membrane ruffles

Which of the following is a difference between endogenous and exogenous antigen presentation? A. MHC-II presents only endogenous antigens. B. Endogenous antigens bind to the TCR of a TH cell. C. Endogenous antigens are always "self," and exogenous are "foreign." D. Endogenous antigens are displayed only on professional APCs. E. Endogenous antigens are processed by the proteasome, exogenous by the phagolysosome

endogenous antigens are processed by the proteasome, exogenous by the phagolysosome

Which of the following is true of endotoxin, but not of an exotoxin? A. Endotoxin is a cytoplasmic protein. B. Endotoxin has much higher toxocity. C. Endotoxin stimulates interleukin release, which results in fever. D. Endotoxin can be the target for a vaccine. E. Diseases caused by endotoxin can rarely be treated with antibiotics

endotoxin stimulates interleukin release, which results in fever

Haemophilus influenzae is a major cause of . . . A. eye, ear, nose and throat infections and meningitis B. lower respiratory tract infections in elderly adults C. influenza pandemics D. skin infections associated with severe acne in teenagers E. mild "walking pneumonia" in young adults

eye, ear, nose, and throat infections and meningitis

Rotavirus is frequently transmitted in daycare centers. Recalling what this virus causes, which of the following is the most likely route of transmission? A. Foodborne B. Vertical C. Direct contact D. Airborne E. Fecal-oral

fecal-oral

Two examples of passive immunity are . . . A. IgA in breast milk and IgA produced in response to a type of flu vaccine B. live attenuated vaccines and inactivated vaccines C. oral vaccines and injected vaccines D. the two components of a tetanus shot E. Fetal IgG protection and Diphtheria immune globulin

fetal IgG protection and Diphtheria immune globulin

Rubella ("German measles") is most serious in . . . A. fetuses B. infants C. teenagers D. young adults E. elderly adults

fetuses

What is pneumonia? A. a bacterial infection that produces a toxin that damages lung epithelial cells B. formation of lung granulomas in response to an infection C. hemorrhage of blood into the lungs caused by septicemia D. lung damage caused by an AB toxin that interferes with cAMP regulation E. fluid secretion into the lungs as part of our immune response to an infection

fluid secretion into the lungs as part of our immune response to an infection

The Latin word 'tinea' means "moth-like," referring to the roughened patches of skin associated with this type of disease. The disease is caused by a ______________ that has _____________ as one of its main virulence factors. A. fungus; keratinase B. fungus; dimorphism C. virus; IgA protease D. bacterium; an AB toxin E. bacterium; coagulase

fungus; keratinase

The Latin word tinea means "moth-like," referring to the roughened patches of skin associated with this type of disease. The disease is caused by a ______________ that has _____________ as one of its main virulence factors. A. bacterium ; an AB toxin B. fungus ; dimorphism C. virus ; syncytial formation D. fungus ; keratinase E. bacterium ; coagulase

fungus; keratinase

Streptococcus and Staphylococcus infections can often resemble each other, but some are quite distinct. Which of the following is exclusively a Staphylococcal disease? A. Erysipelas B. Impetigo C. Furuncles D. Intoxication with a superantigen toxin E. Cat-Scratch Disease

furuncles

Most of the really dangerous pathogenic fungi can do which of the following? A. Get inhaled as mold spores and transition to yeast in the host's body B. Produce spores within the host's body C. Escape the immune system by living inside red blood cells D. Produce endotoxin E. Grow as yeast in an environmental reservoir outside the host

get inhaled as mold spores and transition to yeast in the host's body

Shigella are intestinal bacteria that can break out of the phagosome. What strategy do they use to infect a host? A. Prevent opsonization with antibodies to avoid phagocytosis B. Coat themselves with host proteins to resemble "self" cells C. Use Type III effectors to enter the apical membrane of intestinal epithelial cells D. Get into macrophages as quickly as possible through intestinal M-cells E. Develop a large anti-phagocytic polysaccharide capsule

get into macrophages as quickly as possible through intestinal M-cells

An example of passive immunity is . . . A. getting a DTaP booster shot after you have had the initial vaccine B. getting injected with blood serum from a patient who formerly had diphtheria C. not getting measles because most of the people around you have been vaccinated D. using the nasal "Flu-Mist" spray rather than getting a flu shot E. getting injected with an allergen to help you overcome a childhood allergy

getting injected with blood serum from a patient who formerly had diphtheria

Which of the following is typical in cases of miliary tuberculosis, but not in other TB cases? A. A Type IV hypersensitivity against TB proteins. B. Unexplained weight loss and chronic fatigue. C. Granuloma formation in the liver with eventual liver failure. D. Requirement to treat for months with many different antibiotics. E. Recruitment of lymphocytes and macrophages toward infected cells

granuloma formation in the liver with eventual liver failure

Mycobacterium tuberculosis (Mtb) can prevent phagosome / lysosome fusion. Therefore . . . A. it is very difficult for Mtb to spread person-to-person B. Mtb must be treated with antibodies rather than with antibiotics C. Mtb can activate macrophages without TH cell involvement D. granulomas can form in some cases of Mtb infection E. Mtb can cause liver damage in addition to damage elsewhere in the body

granulomas can form in some cases of Mtb infection

Medical and Nursing students are taught to recognize infections with Pseudomonas aeruginosa as quickly as possible. How and why? A. green pigment; Forms biofilms that are almost impossible to treat B. "Bulls' eye" rash; Forms biofilms that are almost impossible to treat C. green pigment; Immune mimicry leads to autoimmune sequelae D. "Bulls' eye" rash; Immune mimicry leads to autoimmune sequelae E. direct ELISA test; Rapid spread of the infection throughout the body

green pigment; Forms biofilms that are almost impossible to treat

Which of the following is NOT an effective way to reduce the incidence of nosocomial infections? A. have health care workers use masks and eye protection B. glove usage for all patient C. contact use catheters only when necessary D. group patients into rooms irrespective of their health conditions E. have health care workers wash their hands between each patient they touch

group patients into rooms irrespective of their health conditions

Antigenic shift in influenza virus . . . A. happens every year B. happens because influenza has a segmented genome C. is predictable for vaccine manufacture D. happens because the influenza genome is not proofread E. occurs only when influenza infects human

happens because influenza has a segmented genome

The most effective way to reduce the incidence of nosocomially spread infections is to . . . A. Disinfect operating rooms better between surgeries B. Use more antibiotics in hospitals C. Fit HEPA filters in all patient rooms D. Separate patients into wards, grouped by the disease they have. E. Have health care workers wash their hands between each patient they touch

have health care workers wash their hands between each patient they touch

A 7-year-old child presents at a clinic with heartbeat abnormalities, a fever, and a whitish membrane across the back of his throat. How should he be treated? A. He should receive the DTaP vaccine. B. He should receive antibiotics. C. He should receive the antiviral medicine amantadine. D. He should receive a passive IgG injection. E. His symptoms should be treated, but there is nothing that can treat the disease

he should receive a passive IgG injection

What is usually the eventual cause of death in diphtheria patients? A. Heart and kidney damage due to an AB cytotoxin that is absorbed in the blood. B. Septicemia due to an invasive toxin. C. Anemia due to iron binding by a bacterial siderophore. D. Asphyxiation due to a pseudomembrane that blocks the tracheal opening. E. Nephritis due to bacterial colonization in the kidneys

heart and kidney damage due to an AB cytotoxin that is absorbed in the blood

A patient presents himself to a clinic in Connecticut with a concentric, "bulls' eye" rash, muscle aches and low-grade fever. What symptoms would likely be in this patient's future? A. Acute glomerulonephritis B. Cirrhosis and liver cancer C. Heart arrhythmia and joint pains D. Disfiguring tissue granulomas E. Tissue death and gangrene

heart arrhythmia and joint pains

A pathogen that has evolved for a long time in the same environment as its host will exhibit which of the following? A. High prevalence and high mortality B. High prevalence and low mortality C. Low prevalence and high mortality D. Low prevalence and low mortality E. High incidence and low prevalence

high prevalence and low mortality

A pathogen that has recently emerged in a particular region of the world would be expected to have which of the following epidemiological characteristics? A. High virulence and low prevalence B. Low infectious dose and low mortality C. Low prevalence and high incidence D. Low mortality and high prevalence E. Balanced pathogenicity

high virulence and low prevalence

Respiratory disease common in the midwest US spread by contact with aerosols from bird droppings and bat guano. A. Histoplasmosis B. Hantavirus C. Coccidioidomycosis D. Trichonympha E. Cryptococcosis

histoplasmosis

The common cold is most frequently caused by . . . A. Human rhinovirus B. Corynebacterium C. Group A Streptococcus D. Varicella virus E. Adenovirus

human rhinovirus

For which type of bacterial toxin is there a medical use - to make anesthetics more readily able to penetrate through skin and other tissues? A. cAMP-producing A-B toxins B. phospholipases C. IgA proteases D. hyaluronidases E. superantigen toxins

hyaluraonidases

For which bacterial toxins have we found a pharmaceutical use to make anesthetics more readily able to penetrate through skin and other tissues? A. ADP-ribosylating A-B toxins B. hemolysins C. hyaluronidases D. endotoxins E. superantigen toxins

hyaluronidases

Which of the following are produced inside a phagolysosome? A. Hydrolases and Interleukins B. Digestive enzymes and MHC-II C. Peroxides and Antimicrobial complement peptides D. Cytokines and Cytotoxins E. Hydrolases and Peroxides

hydrolases and peroxides

How do you know that a person has been exposed to a particular pathogen within the past year or so? A. You'd have to perform an immunoelectrophoresis test. B. If the well of a direct ELISA microtiter plate turns red C. If the person produces monoclonal antibodies against the pathogen D. If the person's antibody titer against the pathogen, which was 1:8, is now 1:128

if the person's antibody titer against the pathogen, which was 1:8, is now 1:28

What is meant by "cellular" immunity? A. Any immune response involving cells (B cells, Dendritic cells, T cells, etc.) B. Immunity related specifically to T cells or NK cells. C. Any immune response that requires cell division. D. Immunity related specifically to the presence of phagocytes. E. An overactive immune response also known as a Type IV hypersensitivity

immunity related specifically to T cells or NK cells

Which of the following tests DOES NOT use two antibodies, one labeled and one unlabeled? A. Western Blot (Immunoblot) B. Direct ELISA test C. Indirect ELISA test D. Indirect fluorescent antibody test E. Immunodiffusion test

immunodiffusion test

The picture at right shows a bacterium (arrow) entering an epithelial cell. The elaborate structure that surrounds the bacterium forms ... A. when the PAMP on the epithelial cell contacts the TLR on the bacterium. B. by extension of the bacterial cell envelope C. as a defense mechanism to prevent the epithelial cell from being infected D. in response to a bacterial protein secreted by a Type III secretory system E. as the first step in the membrane fusion process

in response to a bacterial protein secreted by a Type III secretory system

A major advantage of using inactivated vaccines rather than live vaccines is that . . . A. inactivated vaccines produce a longer immune memory B. inactivated vaccines stimulate B cells and T cells equally C. inactivated vaccines can be given to immunocompromised patients D. inactivated vaccines are easier to administer to a patient E. live vaccines require an adjuvant to avoid a serious anaphylactic response

inactivated vaccines can be given to immunocompromised patients

Which of the following would be characteristic of a highly evolved pathogen? A. Incidence of infection 50%, mortality 95% B. Incidence of infection 1 in 10,000, mortality 2% C. Incidence of infection 100%, mortality 100% D. Incidence of infection 1 in 10,000, mortality 90% E. Incidence of infection 90%, mortality 1 in 1,000

incidence of infection 90%, mortality 1 in 1,000

In the last 50 years, deaths in the US due to infectious diseases have . . . A. decreased dramatically B. remained about the same each year C. increased again for the last 30 years after initial decrease D. mostly been prevented due to clean food and water laws and increased sanitation E. mostly been prevented due to extensive and widespread use of antibiotics

increased again for the last 30 years after initial decrease

Which of the following best explains the trend in the incidence of infectious diseases in the US since 1980? A. Increased stringency of public health and sanitation laws B. Many diseases evolving towards balanced pathogenicity C. Presence of endemic diseases in certain parts of the country D. Increased use of new, more effective vaccines E. Increased use of antibiotics, especially in nosocomial settings

increased use of antibiotics, especially in nosocomial settings

All of the following are important for bacterial survival in a host EXCEPT the ability to . . . A. digest C5a B. produce siderophores C. bind complement regulatory protein D. induce autoimmune antibodies E. produce an Fc binding protein

induce autoimmune antibodies

Which of the following is correct regarding the body's interferon defense system? A. It can be activated when antibodies bind to an organism. B. Cells that produce interferon apoptose when infected by a virus. C. Interferon induces the production of antiviral proteins in adjacent cells. D. Interferon is only produced by antigen presenting cells (APCs). E. Interferon is produced in the phagolysosome of macrophages with activated TLRs

interferon induces the production of antiviral proteins in adjacent cells

Systemic Lupus Erythematosus . . . A. is a serious bacterial infection of the whole body B. is an autoimmune disease in which your T cells recognize your own proteins C. involves production of antibodies that activate thyroid-stimulating hormone receptors D. is an inherited immunodeficiency disease E. involves formation of immune complexes containing your own DNA

involves formation of immune complexes containing your own DNA

The process responsible for antibody diversity . . . A. puts the Fab and Fc regions together B. joins heavy chains to light chains C. begins as soon as a B cell comes into contact with an antigen D. is initiated by cytokine signals from macrophages E. is a genetic recombination that can only happen once in each B cell

is a genetic recombination that can only happen once in each B cell

Immunoglobulin M (IgM) . . . A. looks like this: >=< B. is the main serum antibody C. is mostly secreted into the tissues and mucus membranes D. is the first antibody produced in response to an infection E. has a longer half-life than any of the other antibodies

is the first antibody produced in response to an infection

Immunoglobulin G (IgG) ______________________. A. never undergoes somatic recombination B. is the main serum antibody C. has four binding sites D. is the first antibody produced in response to an infection E. is mostly secreted into the tissues and mucus membranes

is the main serum antibody

What is the purpose for our immune system to opsonize bacterial cells? A. It makes the bacteria easier for the oxidative burst and hydrolases to digest. B. It causes phagocytes to produce more lysosomes. C. It makes the surface of the bacteria more slippery so that we can excrete them better. D. It adds surface features to the bacterium that makes it easier for our cells to recognize. E. It is a way to neutralize bacteria so that they do not attach to our cells and infect us

it adds surface features to the bacterium that makes it easier for our cells to recognize

What is the function of a Toll-like receptor? A. It allows macrophages and dendritic cells to bind specifically to pathogens. B. It allows TH cells to bind specifically to B cells. C. It allows pathogens to bind specifically to epithelial cells. D. It allows bacteria to bind the FC part of antibodies. E. It allows the immune system to recognize when a viral infection has occurred

it allows macrophages and dendritic cells to bind specifically to pathogens

How is LPS toxic to humans? A. It agglutinates a type of red blood cell, causing DIC. B. It activates B cells even without antigen binding, causing massive antibody release. C. It binds to cells in the hypothalamus, damaging them and causing fever. D. It increases cAMP concentration, causing edema and cell death. E. It binds strongly to macrophages, causing a response that can lead to hypovolemia

it binds strongly to macrophages, causing a response that can lead to hypovolemia

What is the function of the B part of A-B toxins? A. It binds to and inhibits B cells. B. It enters the host cell and damages host cell metabolism. C. It binds to surface receptors on the host cell. D. It causes the main symptoms of anthrax. E. It inhibits phagosome-lysosome fusion

it binds to surface receptors on the host cell

Which of the following is true with respect to tuberculosis? A. DOT is a new antibiotic that has been approved for treatment of XDR-TB. B. Fecal transmission is important in addition to respiratory droplet transmission. C. Nowadays it is typically spread zoonotically to humans by dogs. D. It can affect the bones and liver as well as the lungs. E. It cannot be treated by antibiotics that target the cell wall

it can affect the bones and liver as well as the lungs

One of the complement activation pathways is called the "alternate" pathway. What is different between this pathway and the "classical" pathway? A. It kills bacteria in a different manner than the classical pathway. B. It requires a bacterial surface to be coated with lectins. C. It only kills infected "self" cells, rather than pathogenic bacteria. D. It can be activated even before the humoral immune response is active. E. It alternates between inducing inflammation and keeping fever from getting too high

it can be activated even before the humoral immune response is active

How can IgG prevent a viral infection? A. It can induce ADCC against the virus. B. It can stop the synthesis of viral DNA or RNA. C. It can immobilize the virus in the bloodstream. D. It can neutralize viral binding to host receptors. E. It can direct the insertion of a membrane attack complex into the viral capsid

it can neutralize viral binding to host receptors

Why can't we produce a successful vaccine against endotoxin? A. Our immune response against it is too weak. B. It can't be inactivated because it isn't a protein. C. It resembles human host proteins too closely. D. It enters host cells so quickly that antibodies never encounter it. E. It is found on a plasmid, and can spread too rapidly through a bacterial population

it can't be inactivated because it isn't a protein

A person who is allergic to pollen has small amounts of pollen injected under his skin. What is the purpose of this? A. It reduces the number of anti-pollen antibodies he makes. B. It removes the memory T cells that recognize pollen antigens. C. It reduces the affinity of his mast cells for pollen. D. It "toughens" his immune system, adapting it to anaphylaxis. E. It causes him to make more anti-pollen IgG, rather than IgE

it causes him to make more anti-pollen IgG, rather than IgE

How does a superantigen toxin function? A. It is a very strong antigen that provokes a massive antibody response. B. It crosslinks the TCR of TH cells to MHC2 that is not presenting an antigen. C. It crosslinks TH cells to B cells, thereby producing lots of antibodies. D. It dissolves the material that holds cells together in tissues. E. It adds ADP-ribose to a protein, thereby inactivating it and killing cells

it crosslinks the TCR of TH cells to MHC2 that is not presenting an antigen

How does production of phospholipase enhance bacterial survival inside macrophages? A. It binds the Fc end of IgG, thereby disguising the bacterium as a "self" cell. B. It oxidizes macrophage proteins and kills the macrophage. C. It prevents C3b from binding to the bacterial surface and activating complement. D. It digests the antibody that most commonly attacks bacteria inside macrophages. E. It disrupts the phagosome membrane, allowing the bacterium to escape

it disrupts the phagosome membrane, allowing the bacterium, to escape

What is the function of the A part of an AB toxin? A. It binds to surface receptors on the host cell B. It hydrolyzes the head group of phospholipids C. It facilitates bacterial spread through a tissue and into the bloodstream D. It activates large numbers of TH cells E. It enters the cytoplasm and kills the host cell, often by altering cell metabolism

it enters the cytoplasm and kills the host cell, often by altering cell metabolism

Infection with Pseudomonas is best treated preventatively. It is very difficult to treat once an infection has already occurred. Why? A. It is Gram positive, but most antibiotics only treat Gram negative infections. B. It is highly invasive, spreading throughout the body very rapidly. C. It forms biofilms, which are very difficult to treat with antibiotics. D. It lives inside macrophages, where antibiotics cannot penetrate. E. They type of patients it infects are not able to tolerate most antibiotics

it forms biofilms, which are very difficult to treat with antibiotis

A 20-year-old student developed a cough and mild fever of 4 weeks' duration. The causative organism attached to his lung epithelial cells and caused irritation by secreting H2O2. What is unusual about this organism? A. It has no cell wall. B. It induces autoimmune sequelae. C. It allows formation of syncytia. D. It survives in macrophages and spreads throughout the body. E. It has an RNA genome and so its surface proteins undergo mutational variation

it has no cell wall

The measles virus is considered to cause an immunodeficiency disease because . . . A. it causes you to produce antibodies against your own cells B. it infects T cells and damages your cellular immune system C. it can be transmitted vertically from mother to child D. it occurs mainly in immunocompromised people E. you can develop a Type I hypersensitivity response to it if you are infected more than once

it infects T cells and damages your cellular immune system

What is a hapten? A. It refers to the way the Fc portions of IgM are arranged. B. It is a cluster of antibodies and antigens that triggers a type III hypersensitivity. C. It is a small molecule that binds to a cell and triggers a type II hypersensitivity. D. It can be injected into the blood to prevent a type I hypersensitivity. E. It is a new type of conjugated vaccine that uses the DNA of the pathogen

it is a small molecule that binds to a cell and triggers a type II hypersensitivity

Which of the following statements about antigenic shift is NOT true? A. It is caused by the lack of proofreading by RNA polymerase. B. It requires two influenza A viruses to infect the same host cell. C. It requires the influenza virus to have a segmented genome. D. It affects mainly the Hemagglutinin and Neuraminidase proteins. E. It is the main reason for periodic worldwide flu pandemics

it is caused by the lack of proofreading by RNA polymerase

The average monthly prevalence of a disease in several countries for the period 2004-2008 is shown in the table at right as cases per 100,000 people. What can you say about the disease? A. There was an epidemic of it in Canada in 2005. B. It is likely to become a worldwide pandemic in 2009. C. It was an acute illness in the UK in 2006. D. It is likely a chronic disease in the United States. E. It is endemic in the United States.

it is endemic in the United States

The incidence of a particular disease was followed for a year (graph at right). Which of the following is the most reasonable statement about its epidemiology? A. It is likely spread by direct contact. B. It is likely spread by a biological vector. C. There was a pandemic of the disease in June. D. Its prevalence is much higher than its incidence. E. It appears to be foodborne or waterborne.

it is likely spread by a biological vector

Which of the following correctly refers to the second signal that is required for T cell activation? A. It is secreted by a B cell that has already bound antigen. B. It is a protein that attaches to MHC-II. C. When T cells bind it, they become more energized, anergic T cells. D. It is a cytokine that enhances the antibody-producing ability of T cells. E. It is only produced when a pathogen is recognized by the immune system

it is only produced when a pathogen is recognized by the immune system

What is the role of the B7 protein in the immune response? A. It is a receptor on macrophages that recognizes pathogens. B. It is produced by infected macrophages to help stimulate T cells. C. It is a type of antibody produced by effector B cells. D. It is a protein on TC cells that allows them to bind to TH cells. E. It is secreted by TH cells to stimulate B cell clonal expansion and differentiation

it is produced by infected macrophages to help stimulate T cells

How does interferon fight a viral infection in an infected host? A. It is a viricidal protein. B. It induces antiviral proteins (AVP) in virus-infected cells. C. It boosts the production of B cells in a virus-infected host. D. It activates inactive AVPs in virus-infected cells. E. It is produced in virus-infected cells and induces AVP in neighboring cells.

it is produced in virus-infected cells and induces AVP in neighboring cells

The current vaccine against Streptococcal pneumonia is produced by attaching the capsule polysaccharide from S. pneumoniae to a protein and injecting it into the patient. What purpose does the protein serve? A. It is recognized by TH cells, allowing a stronger B cell response to the vaccine. B. It is an adjuvant that boosts the ability of macrophages to engulf the vaccine. C. It is a platform to make the vaccine antigen easier to manipulate in the lab. D. It allows B cells to respond to the vaccine without the need for T cell involvement. E. It allows the body to produce antibodies against more diverse antigens

it is recognized by TH cells, allowing a stronger B cell response to the vaccine

The incidence of a particular disease was followed for two years (graph at right). Which of the following is the most reasonable statement about its epidemiology based on the graph? A. It is spread by respiratory droplets. B. It is spread by a mechanical vector. C. It probably spreads in a city's water supply. D. It is an emerging disease that is transmitted by fleas. E. It is foodborne and associated with peanut butter.

it is spread by a mechanical vector

A particular disease has a high seasonal incidence in the United States during February. Which of the following is probably true about this disease? A. It is a chronic illness. B. It is propagated as a common-source epidemic. C. Its incidence in February may be high but its prevalence is still low. D. It is spread through respiratory droplets. E. It is transmitted by a vector

it is spread through respiratory droplets

The antibody type shown at right has what special feature? A. It can cross the placenta to protect the fetus. B. It can activate ADCC better than other antibody types. C. It is involved in the Type I hypersensitivity response. D. It is the major antibody found in mucus membranes. E. It is the first antibody produced in response to an infection

it is the major antibody found in mucus membranes

Which of the following is an advantage of administering a vaccine for Typhoid fever orally? A. It leads to the production of IgA, which protects the portal of entry for this infection. B. It produces a stronger cellular immune response than injected vaccines would. C. You can administered a killed vaccine orally without an adjuvant. D. An oral vaccine can replicate in the body, but an injected one cannot. E. The oral route allows a faster switch to IgG than the injected route

it leads to the production of IgA, which protects the portal of entry for this infection

The malaria parasite has a particularly effective method of avoiding both humoral and cellular host immune defenses. Which of the following is correct about this parasite? A. It lives in red blood cells B. It produces keratinase C. It grows within and travels between host epithelial cells D. It coats itself with host proteins so that it resembles "self" E. It causes host cells to fuse into syncytia

it lives in red blood cells

Which of the following statements is true of MHC-II, but not of MHC-I? A. It must be matched in transplanted tissues or the transplant will be rejected. B. It mainly displays antigens from the phagolysosome. C. It typically displays self antigens as well as foreign antigens. D. It is more important for fighting viral infections than bacterial infections. E. It recruits TC cells and attaches specifically to them so the infected APC will be killed

it mainly displays antigens from the phagolysosome

Which of the following statements about influenza is FALSE? A. There is a live vaccine and an inactivated vaccine against it. B. Tens of thousands of Americans die every year from it. C. It has a high mutation rate and is constantly changing. D. It only infects humans, so can potentially be eradicated. E. There is the potential for a new vaccine that recognizes many varieties of it

it only infects humans, so can potentially be eradicated

A naked virus, HPV, produces a protein called E6. Recalling what infection HPV causes, how does the E6 protein contribute to the infection? A. It prevents the action of the p53 tumor suppressor gene in the host. B. It blocks the MHC-I antigen presentation pathway in the host. C. It allows the virus to form syncytia between infected host cells. D. It prevents activation of TH cells by acting like an analog of MHC-II. E. It stimulates the action of NK cells, which cause the HPV skin lesions

it prevents the action of the p53 tumor suppressor gene in the host

Many pathogenic bacteria have evolved a C5a peptidase. What is the function of this virulence factor? A. It reduces the host's ability to recruit phagocytes to the site of infection. B. It binds host antibodies and neutralizes them. C. It prevents the use of the membrane attack complex by the complement system. D. It allows the bacterium to survive inside macrophages. E. It creates membrane ruffles that enhance bacterial uptake by host cells

it reduces the host's ability to recruit phagocytes to the site of infection

Which of the following is true about antigen presentation on MHC class I? A. It requires protein digestion by the proteasome. B. It only occurs in professional APCs. C. It requires altering the route of membrane vesicles through the endomembrane system. D. Only foreign antigens are presented on MHC class I. E. TH cells are stimulated by binding to antigens presented on MHC-I

it requires protein digestion by the proteasome

How does Mycobacterium tuberculosis cause the formation of the structure indicated in the X-ray at the right? A. It hydrolyzes tissue and damages the lung. B. It multiplies rapidly to form the large bacterial colony seen in the X-ray. C. It survives in macrophages, which continue to secrete cytokines. D. It produces a toxin that results in the formation of blood clots. E. It forms syncytia, one of which is seen in the X-ray

it survives in macrophages, which continue to secrete cytokines

Antigenic drift is especially rapid in influenza virus because . . . A. it has a segmented genome B. it infects humans and pigs as well as birds C. it uses RNA as its genetic material D. it is spread via respiratory droplets E. it has two surface proteins that can recombine

it uses RNA as its genetic material

Which of the following would you expect to be the eventual outcome of an evolutionary selection process in a pathogenic bacterium? A. Its ID50 will change gradually from 102 bacteria to 106 bacteria. B. It will lose the proteins at the tips of its adhesion pili. C. It will increase the size of its genome as it acquires a broad metabolic capacity. D. It will change from an acute infection to a chronic one. E. It will acquire the ability to kill its host much more quickly.

it will change from an acute infection to a chronic one

A friend of yours who knows you took Bio 221 is planning to invest in a company that says they are working on an anti-cancer vaccine. The friend asks your advice about his investment. What would you say to him? A. This is a scam. Such vaccines are not possible, based on what we know about cancer cells. B. It's a DNA vaccine designed to boost your TC cell response to cancer-specific antigens C. It involves microinjecting tumors with substances designed to boost NK cell response. D. It's a good idea, but it will only work for the few cancers that are caused by infectious agents like viruses. E. It probably tries to stimulate self-recognizing antibodies that can kill cancer cells.

it's a DNA vaccine designed to boost your TC cell response to cancer-specific antigens

What causes a B cell to become anergic? A. It produces extra ATP by oxidative phosphorylation. B. A TC cell binds to its MHC2 receptors rather than a TH cell. C. Its MHC2 receptors present an antigen that is not recognized by a TH cell. D. Its MHC1 receptors bearing foreign antigens are recognized by antibodies. E. A TH cell binds to it and secretes stimulatory cytokines

its MHC2 receptors present an antigen that is not recognized by a TH cell

The organism that produces the infection shown in the picture has what major virulence factor? A. a lipid envelope B. dimorphic growth C. ability to survive in macrophages D. phospholipase E. keratinase

keratinase

What is run on the gel if a Western blot is being used as a direct immunological test? A. Antibodies from the patient's serum B. Known antibodies C. Antigens from the patient's serum D. Known antigens E. A Western blot can't be run as a direct test.

known antibodies

People born with SCID can't perform somatic recombination. Which of the following best characterizes this condition? A. lack of both cellular and humoral immunity, but normal innate immunity B. lack of antibodies, but normal cellular immune responses C. inability to delete self-recognizing B-cells or T-cells from the body D. failure to produce memory cells that respond more quickly to a second infection E. lack of cellular, humoral and innate immunity

lack of both cellular and humoral immunity, but normal innate immunity

According to the "hygiene hypothesis" . . . A. you need some good bacteria to fight off the bad ones B. being "too clean" leads to death of your native flora C. lack of exposure to gut flora leads to fewer Treg cells and more autoimmune diseases D. childhood illness leads to a more robust immune response as an adult E. washing hands appropriately is the single best way to reduce disease transmission

lack of exposure to gut flora leads to fewer Treg cells and more autoimmune diseases

Which of the following is the best statement of the "hygiene hypothesis"? A. Effective handwashing could reduce infectious disease transmission by up to 50%. B. Oral vaccination creates a stronger immune response than injected vaccination. C. Lack of personal hygiene leads to many fecal-oral bacterial infections. D. The sicker someone is as a child, the more healthy they will be as an adult. E. Lack of oral exposure to normal flora fails to induce the development of Treg cells

lack of oral exposure to normal flora fails to induce the development of Treg cells

One reason why a vaccine against malaria has proven difficult to design is that the malaria parasite. . . A. has no surface proteins B. has a virus-like protective coating that prevents antibody binding C. covers itself with host antigens D. lives in red blood cells, where it can avoid both cellular and humoral immunity E. can swim through a tissue, avoiding humoral immunity

lives in red blood cells, where it can avoid both cellular and humoral immunity

How could you tell if a particular pathogen produced a superantigen toxin when it infected a patient? A. Look for complete clearing when the pathogen is grown on a Blood Agar plate. B. Look for tissue damage in a patient. Only superantigen toxins can do that. C. Look for the release of cytokines from TH cells that do not recognize the pathogen. D. Look for a larger than normal antibody titer against the pathogen. E. Look for symptoms of DIC. Any pathogen that causes DIC probably produces a superantigen toxin.

look for the release of cytokines from TH cells that do not recognize the pathogen

Which of the following is NOT a morphological or physiological feature of the respiratory system and associated organs that protects it against infection by pathogens? A. ciliated epithelial cells B. low pH C. mucus secretion D. lysozyme E. native bacterial flora

low pH

Which of the following is a type of cell involved in the immune response? A. B cell receptor B. interleukin-2 C. macrophage D. PAMP E. CD4

macrophage

The United States recently switched from the DPT vaccine with a DT booster to the DTaP vaccine with a DTaP booster. What is one reason why this change was made? A. DTaP is an oral vaccine rather than injected. B. Many adults were found to lack immunity to pertussis, so it was added to the booster. C. There was an alarming increase in the prevalence of Diphtheria in the US. D. The 'aP' in the DTaP vaccine provides broader coverage than the 'P' in DPT. E. The DTaP vaccine provides passive as well as active immunity

many adults were found to lack immunity to pertussis, so it was added to booster

What type of cells do NK cells kill? A. any cell with MHC-I on its surface B. T cells that have recognized "self" antigens during development C. B cells without B7 on their surface D. Macrophages that have phagocytized bacteria E. mainly cancer cells and virally-infected cells

many cancer cells and virally-infected cells

Which of the following is an example of an acquired immunodeficiency disease? A. Type I diabetes B. Measles C. Rheumatoid arthritis D. SCID E. Lupus

measles

One advantage of a live attenuated vaccine (LAV) over an inactivated subunit vaccine (ISV) is . . . A. the LAV is more stable under harsh environmental conditions B. the LAV is safe to use in people who are taking cyclosporin or basiliximab C. memory T cells are produced in response to the LAV D. an adjuvant can be used with the LAV to improve B-cell response E. the LAV is quicker to make; the ISV requires generations of growth outside a host

memory T cells are produced in response to the LAV

Which of the following has been most responsible for the trend in infectious disease incidence since 1980? A. Increased availability of clean water in developing countries B. More global travel for tourism and economic migration C. Increased use of vaccines against a larger number of pathogens D. The eradication of many pathogens from the world population E. Failure to comply with electronic disease surveillance programs

more global travel for tourism and economic migration

Which of the following is a difference between rhinovirus and adenovirus? A. One is a naked virus, the other is enveloped. B. One causes colds, the other pneumonia. C. One is spread via respiratory droplets, the other by direct contact. D. One is acid-sensitive, the other can pass unharmed through the stomach. E. There is an effective commercially available treatment for one, but not for the other

one is acid-sensitive, the other can pass unharmed through the stomach

Why are only a very small number of B cells activated in response to an infection? A. Only a few B cells produce the B7 protein. B. Only a few B cells have PMPs that recognize a pathogen. C. Only a few B cells bind to both the specific antigen and a TH cell. D. The cytokines the body produces bind specifically only to one type of B cell. E. Most B cells are activated initially, but the ones that don't respond to the infection are later inactivated again

only a few B cells bind to both the specific antigen and the TH cell

Interestingly, though food proteins are foreign, we usually do not have hypersensitivity reactions against them. What do we think is the reason for this? A. Oral presentation of antigens somehow stimulates regulatory T cells. B. Most food proteins are denatured, and thus don't elicit an immune response. C. We produce enough IgG against them to out-compete the IgE. D. Food proteins only encounter IgA, which produces a very weak immune response. E. There are special cells in the gut that recognize "anti-PAMPs" in food

oral presentation of antigens somehow stimulates regulatory T cells

A patient has an upper respiratory infection caused by Haemophilus influenzae. It would not be unusual if the patient developed __________ within the next week. A. pneumonia B. otitis media C. endocarditis D. flu E. pertussis

otitis media

Giving a shot of tetanus toxoid antibodies after a patient has been infected with tetanus is an example of. . . A. active, artificial immunity B. passive, artificial immunity C. active, natural immunity D. passive, natural immunity E. a conjugate vaccine

passive, artificial immunity

Which of the following is NOT a consequence of activating the complement cascade on the surface of a bacterium? A. Peptide C3b attaches to the bacterium, making it easier to phagocytize. B. Peptide C5b recruits a pore complex to kill the bacterium. C. Convertases are assembled that proteolyze other complement proteins. D. Peptide C3a induces B cells to produce antibodies that attach to the bacterium. E. Peptide C5a recruits PMNs to the site of the infection, where they engulf bacteria

peptide C3a induces B cells to produce antibodies that attach to the bacterium

Which of the following is required for granuloma formation? A. production of antibodies against a bacterium B. epithelial cell damage C. persistent antigen presence inside macrophages D. binding of the B7 on APCs to CD28 on TC cells E. high titer of a pathogen circulating in the blood

persistent antigen presence inside macrophages

Effector TH cells can do all of the following EXCEPT . . . A. Phagocytize nearby bacteria B. Activate TC cells that have bound to an MHC1-Antigen complex C. Induce B cell clonal expansion D. Induce the formation of memory T cells E. Cause macrophages to become more potent

phagocytize nearby bacteria

Which of the following is generally the LEAST effective type of vaccine? A. Live attenuated vaccine B. Protein conjugate vaccine C. Subunit vaccine D. Polysaccharide vaccine E. Whole-cell inactivated vaccine

polysaccharide vaccine

Which of the lines in the following table is INCORRECT? TH cells TC cells A. surface antigens CD4, CD28 CD8, CD28 B. binds to . . . MHC-II MHC-I C. presents antigens from cytoplasm cytoplasm and phagolysosome D. secretes interleukins perforins E. antiviral role stimulatory kills infected cells

presents antigens from cytoplasm, cytoplasm and phagolysosome

All of the following factors would presumably be especially important in nosocomial disease transmission EXCEPT . . . A. ability of an organism to form biofilms B. presence of R-plasmids C. transmission by direct contact D. prevalence of an organism in a zoonotic reservoir E. airborne transmission

prevalence of an organism in a zoonotic reservoir

For a chronic disease that is transmitted poorly between hosts (e.g. HIV) . . . A. prevalence would be only slightly higher than incidence B. incidence would be only slightly higher than prevalence C. prevalence would be MUCH higher than incidence D. incidence would be MUCH higher than prevalence E. incidence would always be very low in a population

prevalence would be MUCH higher than incidence

Anthrax spores can survive for many years in soil. Soil is a ______ for anthrax. A. host B. vector C. reservoir D. fomite E. portal of entry

reservoir

Upon receipt of cytokine signals from effector TH cells, macrophages . . . A. undergo class switching and begin to secrete antibodies. B. become antigen-presenting cells. C. differentiate into long-lived memory macrophages. D. produce a more potent oxidative burst that includes nitric oxide. E. begin recruiting the membrane attack complex to kill invading bacteria

produce a more potent oxidative burst that includes nitric oxide

Pharmaceutical companies are finally beginning to develop DNA-based vaccines. These vaccines work by causing a vaccinated person to . . . A. make antibodies against a pathogen's DNA B. produce one of a pathogen's proteins in order to elicit a T cell response C. be protected against pathogens into which the person's DNA has been cloned D. develop immune tolerance to common antigens, thereby preventing allergies E. produce a monoclonal antibody rather than the less effective polyclonal kind

produce one of the pathogen's proteins in order to elicit a T cell response

Bacteria respond to phagocytosis by . . . A. forming granulomas B. releasing their capsule antigens C. producing leukocidins D. enhancing motility to swim out of the phagocyte E. producing an oxidative burst to kill the phagocyte

producing leukocidins

Viruses use all of the following strategies to avoid the immune system EXCEPT . . . A. production of an IgA protease B. production of an interferon repressor C. formation of syncytia D. inhibition of the tumor suppressor p53 E. reversing the MHC-I antigen presentation pathway

production of an IgA protease

Which of the mechanisms below can bacteria use to escape phagocytosis? A. Enhanced motility to get away from phagocytes B. Extra iron binding proteins C. Production of lysins to lyse the phagocyte D. Producing an IgG protease E. Preventing cytokine secretion from T cells

production of lysins to lyse the phagocyte

The three complement activation systems share many common features, but there are also differences. Which of the following is NOT common to all three complement pathways? A. Diapedesis is induced by peptide C3a B. Properdin is required to stabilize the C5 convertase C. C3b opsonizes foreign cells for phagocytosis D. A pore complex is assembled and inserted into a foreign cell to lyse it E. PMNs are recruited to the site of complement activation by peptide C5a

properdin is required to stabilize the C5 convertase

In the past 130 years we have learned quite a bit about microbes. Some things we have learned have required us to modify the system Robert Koch developed in 1880 to identify the causative agent of a disease. Which of the following has NOT required such a modification? A. The recognition that we can only culture about 1% of the bacteria we've discovered B. The discovery of infections caused by biofilms involving several bacterial species C. Realizing that some diseases are of an inflammatory rather than an infectious nature D. Recognizing the concept of idiopathic cases of an infectious disease E. The development of cell culture models to study disease processes at the cellular level

realizing that some diseases are of an inflammatory rather than an infectious nature

What is the most correct statement about the selection process B cells must undergo before they are released to the blood? A. Recognizing any antigens in bone marrow causes B cells to apoptose. B. B cell receptors must recognize self MHCs, but must not recognize antigens on the MHCs. C. Failure to recognize a circulating antigen causes B cells to become anergic. D. V-D-J combinations do not occur if they would produce antibodies against self antigens. E. B cells fail to multiply unless they recognize a self antigen on an MHC2

recognizing any antigens in bone marrow causes B cells to apoptose

Which of the following vaccine types produces the strongest IgA response? A. Toxoid vaccine B. Live, injected vaccine C. Formalin-killed vaccine D. Recombinant, edible vaccine E. DNA-based vaccine

recombinant, edible vaccine

The Varicella-Zoster virus . . . A. remains dormant in nerve tissue and re-erupts after many years. B. may cause encephalitis and pneumonia in addition to swollen parotid glands. C. is the main cause of genital herpes. D. causes an infection that can be treated with tetracycline. E. can form a granuloma and break out to spread to other organs

remains dormant in nerve tissue and re-erupts after many years

The most common immune disorder in the US is selective IgA deficiency. People who suffer from this will be more likely to get ______________ than other people. A. allergies B. systemic lupus erythematosis C. blood-borne illnesses D. viral infections E. respiratory and digestive illnesses

respiratory and digestive illnesses

The most common nosocomial infections are. . A. respiratory infections due to Staphylococcus transmitted in aerosols B. surgical site infections from improper skin disinfection before surgery C. urinary tract infections from improperly disinfected catheters D. bacteremias caused by extremely drug resistant bacterial strains E. ear, nose and throat infections like sinusitis or otitis media

respiratory infection due to Staphylococcus transmitted in aerosols

Type I (juvenile-onset) diabetes is caused by which of the following? A. a congenital viral infection B. a Type II hypersensitivity reaction involving maternal antibodies C. self-recognizing TC cells that have not been deleted D. a Type III autoimmune sequela to a bacterial infection E. antibodies that recognize a person's own DNA

self-recognizing TC cells that have not been deleted

When does a B cell undergo the genetic switch to begin producing IgG instead of IgM? A. Never - the switch allows B cells to begin producing IgM instead of IgG. B. before you're born C. as soon as the B cell binds to a pathogen D. only after a second infection with the same pathogen E. several days after the initial infection with a pathogen

several days after the initial infection with a pathogen

A person who developed the skin rash shown at right might have what complication later on? A. She might get a painful skin rash many years later, because this is a latent virus. B. She might get warts. C. This virus can induce autoimmune kidney sequelae. D. If she is pregnant, her child may be born with congenital Rubella syndrome. E. The virus immunocompromses her, and she could get pneumonia or encephalitis

she might get a painful skin rash many years later, because this is a latent virus

Which bacterial invasion and colonization (I/C) factor is correctly matched with its role in an infection? A. siderophore - steals iron from the eukaryotic host B. fimbriae - injected into a host cell and damage its cytoskeleton C. Type III effectors - prevent uptake of bacteria by host cells D. coagulase - alters the membrane structure of eukaryotic host cells E. IgA protease - allows a bacterium to cause bacteremia

siderophore - steals iron from the eukaryotic host

Some viruses like Papillomavirus encode an inhibitor of the eukaryotic protein p53. Why? A. so that they can coat themselves with a "self" protein B. so that they can prevent infected cells from undergoing apoptosis C. to cancel the "kill" signal that NK cells recognize D. to allow them to invade deep tissues beyond the skin E. so that they can escape from the phagosome before it fuses with the lysosome

so that they can prevent infected cells from undergoing apoptosis

What cellular process allows billions of different antibody Fab fragments to be produced from only a few hundred genes? A. Somatic Recombination B. Clonal Selection C. Clonal Expansion D. Class Switching E. Promoter Multiplicity

somatic recombination

What are fimbriae? A. Special pili with adhesive proteins at their tips B. Cells in mucous membranes that uptake bacteria C. A type of bacterial exotoxins D. Bacterial proteins that bind iron in host blood serum E. Dense polysaccharide fibers present in bacterial biofilms

special pili with adhesive proteins at their tips

The bacterium that causes necrotizing fasciitis produces a hyaluronidase toxin. This toxin gives an advantage to the bacterium because it allows the bacterium to . . . A. spread more rapidly between the cells in a tissue B. avoid the humoral immune response C. induce a huge cytokine response in the host D. be internalized more quickly by host macrophages E. break out of the phagolysosome

spread more rapidly between the cells in a tissue

When following certain supplemental precautions, not only the nurse, but the patient also must wear a face mask. Such a patient is also usually in a negative pressure room. What are we trying to prevent with these precautions? A. Transmission of droplet-borne infections from the patient to the nurse B. Spread of an airborne infection from the patient to others in the hospital C. The patient getting a disease carried by visitors from outside the hospital D. Droplet-borne transmission from the nurse to the patient E. The nurse acting as a fomite to spread the disease to others in the hospital

spread of an airborne infection from the patient to others in the hospital

Which of the following is LEAST likely to be a portal of exit for a disease organism? A. Semen B. Urine C. Feces D. Saliva E. Sweat

sweat

All of the following are true about diphtheria EXCEPT . . . A. There is an effective vaccine against it. B. Symptoms are caused by granuloma formation and cytokines. C. It is spread person-to-person via respiratory droplets. D. The expression of a key toxin is repressed by excess free iron. E. Fatality is frequently due to heart damage as well as airway blockage.

symptoms are caused by granuloma formation and cytokines

During some viral infections, infected cells fuse to form___________because_____________. A. giant cells ; these cells are easier for the immune system to recognize B. granulomas ; infected macrophages continue to secrete cytokines C. granulomas ; these "activated" cells have a more potent oxidative burst D. syncytia ; it allows the virus to avoid serum antibodies E. syncytia ; these fused cells cannot produce interferon

syncytia; it allows the virus to avoid serum antibodies

What is meant by the abbreviation ADCC? A. Opsonization of a bacterium to enhance phagocytosis B. Aggregation of bacterial cells to enhance phagocytosis C. Blocking viral spike glycoproteins from binding to host cell receptors D. Activating the classical complement pathway to destroy a pathogen E. Tagging a cell so that NK cells bind to and destroy it

tagging a cell so that NK cells bind to and destroy it

What would be the most effective way to protect a patient against bacterial endotoxin? A. Vaccinate the patient with an endotoxoid vaccine. B. Test to see if intravenous fluids given to the patient coagulate Limulus blood cells. C. Autoclave all intravenous fluids that might be given to the patient. D. Use a Western Blot to screen any intravenous fluids given to the patient. E. Passively immunize the patient against bacterial endotoxin

test to see if intravenous fluids given to the patient coagulate Limulus blood cells

Tetanus and Gangrene are both diseases associated with traumatic tissue injuries. How are they different? A. They are caused by unrelated organisms. B. Tetanus is an anaerobic infection; Gangrene is aerobic. C. Tetanus causes tissue ischemia; Gangrene causes paralysis. D. Tetanus can be treated with passive immunity; Gangrene can't be. E. Tetanus is much more common in the United States than Gangrene is

tetanus can be treated with passive immunity; Gangrene can't be

There are both 13-valent and 23-valent Pneumococcal vaccines. Which provides more effective long-term protection, and why? A. The 23-valent because it provides immunity against more strains of the bacterium B. The 23-valent because it also provides immunity against the PMNs that cause inflammatory damage. C. The 23-valent because it has more electrons in its outer energy level D. The 13-valent because it contains polysaccharide capsule antigens conjugated to proteins E. The 13-valent because it is a live attenuated vaccine rather than a subunit vaccine

the 13-valent because it contains polysaccharide capsule antigens conjugated to proteins

The causative agent of tetanus is an AB toxin. Which of the following is true about it? A. The A part binds to neurons. B. The B part binds to lung epithelial cells. C. The A part inhibits release of a neuroinhibitor. D. The B part ADP-ribosylates a host cell protein. E. It can be inactivated with an antibiotic

the A part inhibits release of a neuroinhibitor

Which of the following is correctly associated with a B cell that has become anergic? A. Class switching is reversed so that IgG becomes IgM. B. The B cell turns into a less active plasma cell. C. The B cell begins to replicate uncontrollably. D. An oxidative burst from the B cell releases energy. E. The B cell displays an antigen that no T cell recognizes

the B cell displays an antigen that no T cell recognizes

How does a bacterial A-B toxin work? A. It digests the host's cell membrane phospholipids. B. The B part blocks protein synthesis; the A part activates cAMP production. C. The B part binds to receptors; the A part enters the host cell. D. The A part activates the B part, which then binds to host cells and kills them. E. The B part protects the A part from attack by the host's immune system.

the B part binds to receptors; the A part enters the host cell

A B-cell recognizes an antigen, but there is no TH cell that also recognizes the antigen. What will happen? A. The B cell will become anergic. B. The B cell will begin producing antibodies. C. The TH cell will secrete cytokines to activate the B cell. D. The B cell will not internalize the antigen. E. The TH cell will kill the B cell

the B-cell will become anergic

Which of the following statements about AB toxins is accurate? A. The B-part of the toxin can inhibit the function of B cells. B. The A-part of the toxin can hydrolyze phospholipid head groups. C. The B-part of the toxin can be used as a vaccine antigen. D. They have a medical use to allow anesthetics to penetrate tissue better. E. They are only produced by Gram positive bacteria.

the B-part of the toxin can be used as a vaccine antigen

What parts of antibodies have great variability due to somatic recombination? A. variable and constant regions B. the Fc part only C. alpha and beta chains D. the heavy chain only E. the Fab part only

the Fab part only

What is the major difference between IgG and IgA? A. They have different variable regions. B. IgA is produced by class switching, IgG by somatic recombination. C. IgG is the first antibody to be produced, IgA is the second. D. IgG is the major secreted antibody; IgA is found mainly in the blood. E. The Fc region of the IgA heavy chain allows dimerization

the Fc region of the IgA heavy chain allows dimerization

Which of the following would typically NOT be true of an attenuated organism? A. It lacks one or more pathogenicity islands. B. Its ID50 is very low. C. It lacks adhesion pili. D. It generally causes a sub-clinical infection. E. You develop a strong immune response against it

the ID50 is very low

What advantage does a protein conjugate vaccine (PCV) have over a polysaccharide subunit vaccine (psv)? A. The PCV induces cellular as well as humoral immunity. B. The PCV allows TH cells to produce antibodies as well as B cells. C. The PCV is essentially a "double" vaccine, with broader coverage than the psv. D. The PCV induces the formation of memory B cells. E. The PCV produces activated macrophages much more frequently than the psv

the PCV induces the formation of memory B cells

What happens when an effector TC cell binds to an antigen on an epithelial cell's MHC1? A. The TC cell kills the epithelial cell. B. The TC cell recruits B cells to the site of the infection. C. The TC cell releases cytokines that activate TH cells. D. The TC cell becomes anergic. E. The epithelial cell produces cytokines to activate the TC cell

the TC cell kills the epithelial cell

What is wrong with the picture at the right? A. TC cells do not bind to APCs. B. APCs have MHC-II, not MHC-I. C. NK cells release the perforins, not TC cells. D. The target of the effector TC cell should be a bacterium. E. The TC cell shown in the drawing should become anergic, not activated.

the TC cell shown in the drawing should become anergic, not activated

Which of the following does NOT happen once TH cells have become activated? A. The TH cells secrete cytokines B. The TH cells activate macrophages C. The TH cells activate B cells D. The TH cells produce MHC2 on their surface E. The TH cells stimulate themselves to divide and differentiate into memory cells

the TH cells produce MHC2 on their surface

A main virulence factor of both Staphylococcus and Pseudomonas is . . . A. a green pigment that oxidizes macrophages B. an IgA protease C. a superantigen toxin D. the ability to survive within phagocytes E. the ability to form dense biofilms

the ability to form dense biofilms

Which of the following is NOT common to all three complement activation pathways? A. The inflammatory response is triggered by C3a and C5a. B. The adaptive immune response is needed for activation of the pathway. C. Complement protein C3b is needed as part of the C5 convertase. D. A molecule from your blood serum must bind the surface of a pathogen. E. The MAC is formed from C5b and C6 - C9

the adaptive immune response is needed for activation of the pathway

Which of the following could be a possible interpretation of the agglutination test shown in circle A? A. The antibodies recognize the organism's surface antigens. B. The antibodies recognize soluble antigens in the serum. C. The antibodies are present in large excess compared to the antigens D. The antibodies do not recognize any antigens in the sample. E. Sample A was incubated with antibody B.

the antibodies recognize soluble antigens in the serum

If a TH cell binds to an MHC on "cell A" that is presenting an antigen, but binds to nothing else, what happens? A. The body assumes cell A is infected; TH releases perforins and granzymes. B. The body assumes cell A is infected; TH activates nearby B cells. C. The T cell is engulfed by the MHC on cell A. D. The body assumes this is a mistake; the TH cell becomes unresponsive. E. The TH cell remains bound to cell A and attracts antibodies in case an infection is nearby.

the body assumes this is a mistake; the TH cell becomes unresponsive

You are working in pharmaceutical research, and you want to make a vaccine against a bacterial polysaccharide slime layer. You know that you will need to make a conjugated vaccine. Which of the following is true about such a vaccine? A. The conjugated protein may be the antigen that is displayed on a B cell's MHC-II. B. A T cell must recognize the polysaccharide part of the vaccine for it to work. C. The conjugated protein allows the vaccine to bind to more than one B cell receptor at the same time. D. Only the polysaccharide component of the vaccine will be endocytosed by the B cell. E. The conjugate protein must be from the same pathogen as the polysaccharide

the conjugated protein may be the antigen that is displayed on a B cell's MHC-II

Which of the following is true with respect to tuberculosis? A. MDR-TB is the most highly antibiotic-resistant form of the disease. B. The disease can be spread by aerosols in addition to respiratory droplets. C. It is a zoonosis spread to humans by cattle. D. The incidence of the disease is much higher than its prevalence. E. It can affect the bones and liver as well as the lungs

the disease can be spread by aerosols in addition to respiratory droplets; it can affect the bones and liver as well as the lungs

What does it mean for a disease to be a "notifiable" disease? A. The signs and symptoms are so obvious that the disease can be diagnosed quickly. B. The symptoms are so painful that you know right away when you get sick. C. The disease is serious enough that each case must be reported to the authorities. D. It is unusual enough that new cases are often reported in medical journals. E. It spreads so quickly that new cases must be placed in quarantine

the disease is serious enough that each case must be reported to the authorities

A microbiologist tells you that there is no model organism for the disease he is studying. What does he mean by this? A. The structure of the disease organism is difficult to represent with a model. B. There is no single organism that causes the disease. C. The disease is an intoxication rather than an infection. D. He will have to rely on Koch's postulates to study it. E. The disease only affects humans

the disease only affects humans

What is this diagram depicting? A. The typical activation of a TH cell by a macrophage B. The function of a superantigen toxin C. The function of an AB toxin D. The attachment of a bacterium to a T cell E. The activity of Staphylococcal protein A

the function of a superantigen toxin

Which of the following is usually true of an exotoxin, but not true of endotoxin? A. There is a serum binding protein that binds to it. B. It is an integral part of the cell envelope C. It is easily detected by the Limulus amoebocyte assay D. It usually causes a fever E. The genes encoding it can be passed among cells on a pathogenicity island.

the genes encoding it can be passed among cells on a pathogenicity island

Which of the following statements is accurate when comparing Anthrax toxin to LPS? A. LPS is more likely to be inactivated by heat or formalin. B. LPS is probably more toxic per microgram than Anthrax toxin. C. The genes for Anthrax toxin are more likely to be located on a plasmid. D. LPS contains more protein components than Anthrax toxin. E. Anthrax toxin is more likely to remain attached to the bacterial cell membrane

the genes for Anthrax toxin are more likely to be located on a plasmid

A new virus is discovered. It is airborne, has a reservoir in geese, has a high ID50 for humans, has a segmented genome, and has not yet evolved balanced pathogenicity. Which part of this information is good news, and why? A. Its being airborne. You can avoid it by staying a meter away from other people. B. The high ID50. At least it takes a lot of it to kill you. C. Its not being a balanced pathogen. At least it has not yet evolved to become as deadly as it could be. D. Its segmented genome. That allows you to break it up more with a vaccine. E. Its reservoir in geese. That makes it potentially easy to eradicate.

the high ID50. At least it takes of a lot of it to kill you

In nursing, Standard Precautions are taken to prevent transmitting disease via the nurse. In addition, there are Supplemental Precautions based on . . . A. the age of the patient (infant, elderly, etc.) B. the part of the body the nurse must come in contact with C. the mode of transmission of the disease the patient has D. the prevalence of the disease in a particular hospital E. how frequently the nurse must come in contact with the patient

the mode of transmission of the disease the patient has

Which of the following statements about antibody types is TRUE? A. Most B-cells produce IgG as soon as they detect an infection. B. IgA can cross the placenta to protect the baby for a few months after birth. C. IgE is produced by class switching of the genes that encode the Fab part of IgM D. The most effective antibody for agglutinating antigens is IgM. E. The role of IgD is not clear, but it is involved in the allergic response

the most effective antibody for agglutinating antigens is IgM

Which of the following is a similarity between Type I and Type IV hypersensitivity reactions? A. the time it takes for the reactions to develop B. the immune system cells that mediate the reaction C. the need for prior exposure to the antigen that causes the reaction D. the preferred treatment for the symptoms of the reaction E. the type of antibodies involved in the reaction

the need for prior exposure to the antigen that causes the reaction

Some bacteria have evolved the ability to prevent lysosome fusion to a phagosome. These bacteria can therefore avoid . . . A. phagocytosis B. granuloma encasement C. neutrophil attacks D. opsonization E. the oxidative burst

the oxidative burst

The malaria parasite spends most of the human part of its life cycle in red blood cells (RBCs). What does this imply about the body's response to the presence of this parasite? A. Antibodies can't bind to the parasite, but TC cells can still kill infected RBCs. B. Infected RBCs can only be killed by NK cells. C. The parasite can be neutralized by antibodies once the antibodies bind to RBCs. D. The parasite is exposed to neither humoral nor cellular immune responses. E. The parasite's surface proteins resemble those of the host.

the parasite is exposed to neither humoral nor cellular immune response

Two things happen once a pathogen binds specifically to a dendritic cell (DC). They are . . . A. The pathogen is engulfed, and the DC produces co-stimulatory molecules. B. The DC stimulates the production of both antibodies and T cells. C. The DC produces both B7 proteins and PRR proteins. D. The pathogen is engulfed, and the DC begins to produce antibodies. E. The DC begins to coat itself with MHC-I and with PRR proteins

the pathogen is engulfed, and the DC produces co-stimulatory molecules

A patient comes to your clinic with the structures shown at right on his vocal cords. You take a biopsy of the structure indicated by the arrow. PCR primers are prepared comple-mentary to the 3' ends of a bacterial gene, and a reaction is performed on the biopsied tissue. The results are shown after electrophoresis. What is the best interpretation of this data? A. The PCR has been done wrong. One primer should be complementary to the 3' end and the other should be complementary to the 5' end of the gene. B. The patient's macrophages are in the process of digesting the bacteria and releasing the DNA. C. The patient has a granuloma, and may need long term antibiotics to treat it. D. The patient has a tumor, which was caused by insertion of bacterial DNA. E. The bacterium is actively replicating its DNA, which was detected by the PCR

the patient has a granuloma, and may need long term antibiotics to treat it

A patient is suspected of having been infected with Norovirus, which is an acute viral illness. The patient's blood serum is tested by indirect ELISA, and tests positive with a titer of 1:64. This compares with a pre-immune serum testing positive with a titer of 1:4. Interpret the result. A. The patient has been infected with Norovirus. B. The patient is currently infected with Norovirus. C. The patient has not been infected with Norovirus. D. There is a mistake - a direct ELISA should have been used for this. E. There is a mistake - the two titers have been switched

the patient has been infected with Norovirus

What can you conclude if you find the structure at right in a patient? A. The patient has been recently infected. B. The patient is a fetus. C. The patient is suffering from allergies. D. This is at least the second time the patient has had the same disease. E. The patient is suffering an autoimmune disease

the patient has been recently infected

When we say that a patient has a "disseminated" bacterial infection, what can we assume? A. It was transmitted in sexual secretions as an STI. B. The patient has or has had bacteremia. C. The patient is suffering from an autoimmune sequela. D. The infection is transmitted by the airborne route. E. It is an infection of the disseminate, an accessory part of the digestive system

the patient has or has had bacteremia

A patient is suspected of having been infected with an acute viral illness (AVI). The patient's blood serum is tested for antibodies, and tests positive at a titer of 1:16. This compares with a preimmune serum testing positive at a titer of 1:8. Interpret the result. A. The patient has been infected with AVI. B. The patient is currently infected with AVI. C. The patient has not been infected with AVI. D. The patient currently has no circulating antigens from AVI in his blood. E. The test is not valid. The preimmune control test should be negative

the patient is currently infected with AVI

A patient is tested for Hepatitis E by Western Blot. The proteins in the patient's blood plasma are separated by electrophoresis as shown in panel A. After transfer to filter paper, anti-HepE antibodies are added, followed by secondary antibodies (panel B). What can be concluded from the results? A. The patient is not now infected with HepE, but has been in the past. B. Only two bands show up on the blot. The patient does not have HepE. C. The patient does not have HepE, but does have a secondary infection. D. The anti-HepE antibody was a monoclonal antibody. E. The patient is currently infected with HepE

the patient is now not infected with HepE, but has been in the past

Some allergies can be overcome by injecting small amounts of the allergen repeatedly into the patient. How does this work? A. The allergen prevents the patient from making IgE, so no allergic response can occur. B. The allergen activates cellular immunity, which counteracts the allergic response. C. The allergen binds to the surface of mast cells, preventing IgE from binding. D. Injected allergen forms smaller immune complexes, which are non-allergenic. E. The patient produces IgG, which binds the allergen instead of the IgE on mast cells

the patient produces IgG, which binds the allergen instead of the IgE on mast cells

A promising new vaccine technology involves injecting DNA from a pathogen into a patient's cells. How does such a vaccine work? A. The patient's B cells are activated more strongly than with a normal vaccine. B. The patient's MHCs display translated foreign proteins, activating cellular immunity. C. The patient makes antibodies against the pathogen's DNA. D. The DNA is used to reproduce the pathogen inside the patient, activating the same immune response as a direct infection. E. The DNA activates IgA selectively, providing immunity against respiratory pathogens

the patient's MHC's display translated foreign proteins, activating cellular immunity

Antigens displayed on MHC class II come from . . . A. the cytoplasm of the cell that displays them B. the endoplasmic reticulum of the cell that displays them C. the surface of T-lymphocytes that have engulfed bacteria D. the phagolysosome of the cell that displays them E. the endoplasmic reticulum of APCs, but the cytoplasm of other cells

the phagolysosome of the cell that displays them

A man returning from Russia developed a high fever with throat inflammation and a dirty grey membrane across the back of his throat. The doctor told him that antibiotics wouldn't help much with this infection, but he gave the patient a shot anyway. Why? A. A common co-infection can be treated with antibiotics. B. The shot is a vaccine that can prevent further damage. C. The infection might be Strep throat, which needs to be treated. D. The shot is an immune globulin that can block the action of a toxin. E. In case the doctor was wrong and antibiotics could help

the shot is an immune globulin that can block the action of a toxin

Which of the following is NOT true of the chickenpox virus? A. It is very serious if acquired by a fetus. B. It can be spread even before the rash develops. C. Once infected, the host retains the virus for her whole life. D. There is an effective vaccine against it. E. The spots in the rash all look the same a week after the onset of the disease

the spots in the rash all look the same a week after the onset of the disease

A patient is suspected of having been infected with Norovirus, which is an acute viral illness. The patient's blood serum is tested by direct ELISA, and tests positive at a titer of 1:16. This compares with a pre-immune serum testing positive at a dilution of 1:8. Interpret the result. A. The patient has been infected with Norovirus. B. The patient is currently infected with Norovirus. C. The patient has not been infected with Norovirus. D. The patient currently has no circulating antigens from Norovirus in his blood. E. The test is not valid - an indirect ELISA should have been used for this

the test is not valid - an indirect ELISA should have been used for this

There are three complement activation pathways. How do these pathways differ? A. The way the membrane attack complex is assembled B. The way C3 and C5 proteins are hydrolyzed C. The cytokines produced once the pathways have been activated D. The cells that activate the pathway. E. Some pathways, but not all, trigger an inflammatory response

the way C3 and C5 proteins are hydrolyzed

You are a physician examining the blood test of a couple planning to get married. At the right is an agglutination test performed with anti-Rh antibodies. This particular couple belongs to a religion that does not accept transfusion or injection with human blood products. What would you advise them? A. There is a blood mismatch. They should not get married. B. The incompatibility can be overcome by an injection that does not involve blood products. C. The incompatibility will limit them to having only one child. D. There is a blood mismatch, but it won't cause any problem having children. E. There is no blood mismatch

there is a blood mismatch, but it won't cause any problem having children

Diphtheria toxin is an AB toxin. What does this mean? A. It can be detected by the limulus amoebocyte assay. B. There can be no vaccine made against it. C. There is a receptor for it on a host cell membrane. D. Diphtheria is mainly an intracellular pathogen. E. It binds to and alters the structure of antibodies

there is a receptor for it on a host cell membrane

"German" Measles and Measles infections are mostly different, but there are some similarities. Which of the following is similar between these two viruses? A. They are both transmitted by respiratory droplets. B. They both replicate in T cells, reducing the body's ability to fight off other infections. C. They are both associated with a congenital syndrome if they infect a fetus. D. They are both in the same viral group (Rubella viruses). E. Both are associated with small white spots at the site where the virus enters the body

they are both transmitted by respiratory droplets

Which of the following is true regarding interferon alpha and beta? A. They are released from a cell to bind to receptors on nearby cells. B. Their production is induced by viral capsomeres. C. They kill the cell that produces them. D. They are produced in cells next to ones that have been infected by a virus. E. They kill a virus by allosterically inhibiting viral replication enzymes

they are released from a cell to bind to receptors on nearby cells

In rheumatoid arthritis, both B- and T-cells recognize the auto-antigen collagen, and circulating antibodies are produced against collagen. Which of the following is caused by those antibodies? A. They bind to mast cells and elicit a Type I hypersensitivity reaction. B. They bind to collagen and are deposited as immune complexes in joints. C. They bind to collagen and recruit NK cells to destroy collagen cells. D. They also recognize heart muscle, and cause rheumatic fever. E. They recruit TC cells and cause a Type IV hypersensitivity rash

they bind to collagen and are deposited as immune complexes in joints

What do tetanus and gangrene have in common? A. They are both caused by A-B neurotoxins. B. They both require anaerobic conditions for the bacteria to grow. C. They have very similar symptoms, including tissue necrosis. D. Both are generally treated by amputation of the affected body part. E. There is an effective vaccine for both of them

they both require anaerobic conditions for the bacteria to grow

What is the role of antibodies in the innate immune response? A. They can activate the B-cell response. B. They can activate the synthesis of antimicrobial peptides. C. They can detect damage to the skin. D. They can induce the proliferation of myeloid cells. E. Antibodies are adaptive, and have no role in innate immunity.

they can activate the synthesis of antimicrobial peptides

How do live attenuated vaccines produce a strong immune response in a vaccinated host? A. They can be conjugated to proteins. B. They are added with an adjuvant, alum. C. The most highly antigenic subunits can be combined in the vaccine. D. They can replicate in the host and persist for a long time. E. They activate B cells, which produce a stronger response than T cells

they can replicate in the host and persist for a long time

Pharmacologists have learned that conjugated vaccines are much more effective than their polysaccharide vaccine counterparts. What is so special about conjugated vaccines? A. They cause a single B cell to produce multiple different antibodies. B. They contain a protein epitope that blocks an antigen from binding to multiple B cell receptors simultaneously. C. A B cell exposed to such a vaccine can bind to two different antigens simultaneously. D. Two different B cells must participate in the immune response, thereby producing twice the antibodies. E. If a B cell cannot recognize a particular antigen, a conjugate vaccine allows a T cell to recognize it instead.

they contain a protein epitope that blocks an antigen from binding to multiple B cell receptors simultaneously

How do activated macrophages differ from non-activated (naïve) macrophages? A. They contain more lysosomes. B. They can activate B cells. C. They can bind to TH cells. D. They produce more cytokines. E. They have contacted a TC cell to become "activated."

they contain more lysosomes

Which of the following is true about T cells that recognize self antigens? A. They never form. B. They are responsible for Type III hypersensitivity reactions. C. They can activate macrophages even in the absence of protein B7. D. They form because V-J joining is random, but they apoptose in the thymus. E. The second signal that activates them is antibody binding to MHC-II on their surface.

they form because V-J joining is random, but they apoptose in the thymus

Which of the following is a method for bacteria to evade the host immune defenses? A. They use flagella to move from cell to cell . B. They have capsules to avoid phagocytosis. C. They bind the C3b protein, stealing it from the host. D. They block transport of iron, a major oxidant. E. They bind the Fab arms of antibodies

they have capsules to avoid phagocytosis

All of the following are characteristic of bacterial A-B toxins EXCEPT . . . A. The A part can cause increased production of cAMP B. Multiple B parts can bind to multiple receptors C. The B part can be used to produce a toxoid vaccine D. An "attenuated" bacterium might have a mutation in the A part E. They hydrolyze the A part of phospholipids away from the B part

they hydrolyze the A part of phospholipids away from the B part

How do NK cells function? A. They induce apoptosis in any cell to which they bind. B. They kill cells to which they bind unless inhibitory signals outweigh stimulatory ones. C. They bind to antibodies and enhance the ability of antibodies to kill foreign cells. D. They interact with B7 and CD28 to stimulate APCs to kill bacteria. E. They stimulate complement activation by binding to the Fc part of an antibody

they kill cells to which they bind unless inhibitory signals outweigh stimulatory ones

How do antibodies actually kill foreign bacteria? A. They bore a hole in the bacterial cell envelope, making the contents leak out. B. They surround and engulf the bacterium. C. They opsonize the bacterium, making it easier to phagocytize. D. The Fab part directs a protease to the bacterium, which is then digested. E. They anergize the bacterium, causing it to use so much energy that it dies

they opsonize the bacterium, making it easier to phagocytize

What is the role of NK cells in the immune response? A. They can activate the complement system B. They destroy TC cells that recognize self antigens C. They secrete cytokines that activate B cells D. They are responsible for production of T-independent antibodies E. They participate in antibody-dependent cellular cytotoxicity

they participate in antibody-dependent cellular cytotoxicity

Bacteria that can survive inside the phagolysosome can do so because . . . A. they fail to bind C3b to their surface B. they produce leukocidins C. their surface molecules are covered by a capsule D. they produce antioxidants E. they recruit neutrophils to neutralize the phagolysosome

they produce antioxidants

What is the role of Treg cells in the immune response? A. They produce the cytokine IL-10 to reduce the effectiveness of TH and TC cells. B. They produce the cytokine IL-2 to stimulate TC cell differentiation. C. They regulate B-cell clonal expansion in the absence of nearby macrophages. D. They regulate the interaction between TC cells and NK cells. E. They secrete regulatory antibodies that limit the body's autoimmune responses

they produce the cytokine IL-10 to reduce the effectiveness of TH and TC cells

Which of the following is a method for viruses to evade the host immune defenses? A. They use the host's cytoskeleton to spread intercellularly. B. They inhibit their binding to cell surface receptors. C. They produce an antiphagocytic lipid capsule. D. They prevent recruitment of the MAC by the complement cascade E. They produce their own "cancel kill" signal for NK cells

they produce their own "cancel kill" signal for NK cells

How are bacterial type III secretion system effectors involved in causing disease? A. They rearrange the host cytoskeleton and allow the bacterium to enter host cells. B. They allow the bacterium to compete with the host for binding iron. C. They are leukocidins that the bacterium uses to kill phagocytes. D. They mimic "cancel kill" signals so NK cells don't kill infected host cells. E. They bind to host cell receptors and are internalized like toxins

they rearrange the host cytoskeleton and allow the bacterium to enter host cells

In a trial for a vaccine against a respiratory illness that is spread person-to-person, all 1800 adults in a Swiss village were vaccinated with a live attenuated vaccine. 150 of the 1800 people in the trial failed to produce IgG in response to the vaccine, yet they still didn't get sick. Why not? A. They were protected by IgM. B. They were protected by natural, passive immunity. C. They had exceptionally strong immune systems. D. They were protected by herd immunity. E. Their dendritic cells could not recognize the pathogen's PMP.

they were protected by herd immunity

Upon looking at over-the-counter (OTC) medicines for "cold" and "cold with fever" you find that the medicines are pretty much the same. What can you say about this, given your knowledge of the organisms that cause these conditions? A. They're both rhinoviruses, so the treatment should be the same. B. They're very different viruses, but there is no specific treatment for either one. C. The fever is probably a secondary bacterial infection, which can't be treated OTC anyway. D. The treatment for "cold with fever" should be stronger to prevent the cold from turning into Strep throat. E. There shouldn't be an OTC treatment for "cold with fever" - you need to see a Dr.

they're very different viruses, but there is not specific treatment for either one

A patient is infected with Pasteurella multocida from a dog bite. Two weeks later, the same patient is bitten and infected with P. multocida again. The antibody titers were determined after the two bites, and found to be as shown. What best explains this result? A. This patient cannot develop memory B cells. B. This patient has no TH cells. C. The bacteria have not been engulfed by phagocytes. D. TC cells have failed to completely clear the first infection. E. This is the typical result. No special explanation is needed

this patient cannot develop memory B cells

If you are an Rh(-) woman and your husband is Rh(+), why do you need a RhoGAM shot, even for your first pregnancy? A. You could be allergic to your baby's blood. B. To avoid your B cells coming in contact with the Rh antigen. C. So your antibodies don't kill the baby's blood cells. D. To block the formation of immune complexes that may damage the baby's capillaries. E. You don't. You need RhoGAM if you are Rh(+) and your husband is Rh(-)

to avoid your B cells coming in contact with the Rh antigen

Which of the following is true for a direct ELISA, but not for an indirect ELISA? A. To begin the test, known antibodies are bound to the bottom of a microtiter well. B. The test uses an enzyme reaction to visualize antibodies. C. Anti-FC secondary antibodies are used to detect the primary antibodies. D. The test can be used to detect a latent infection. E. The test can be made more sensitive by conjugating latex beads to the known antigen.

to begin the test, known antibodies are bound to the bottom of a microtiter well

A patient is injected with antibodies against tetanus toxin. What is the purpose of this injection? A. To induce a T-dependent response in the patient rather than a T-independent response B. To activate the patient's memory B cells, thereby boosting antibody titer C. To bind and immediately inactivate tetanus toxin in a life-threatening intoxication D. To prevent the patient from making IgE against tetanus toxin E. To enhance the herd immunity level of the community in which the patient lives

to bind and immediately inactivate tetanus toxin in a life-threatening intoxication

What is the purpose of a RhoGAM shot, as given to a pregnant Rh(-) woman? A. To compete with her B cells for binding to fetal red blood cells B. To direct NK cells to kill the mom's B cells C. To out-compete IgE so that the mom doesn't get sensitized to Rh(+) red blood cells D. To prevent the fetal red blood cells from crossing the placenta E. To bind to the mom's B cells and block the binding of Rh antigen

to compete with her B cell for binding to fetal red blood cells

To what can a Western blot be most accurately compared? Why? A. to a direct ELISA since both use secondary antibodies that recognize IgG Fc B. to a precipitin test since both involve agglutination of antigens C. to immunoelectrophoresis, since both involve using antibodies to detect separated antigens D. to a direct fluorescent antibody test, since both involve transfer to filter paper membranes E. to an indirect ELISA since both require antibodies to be bound to a solid support

to immunoelectrophoresis, since both involve using antibodies to detect separated antigens

Neisseria gonorrhoeae (Ng) can bind a host's C3b regulatory protein. Why does Ng do that? A. to inactivate C3b when it binds to Ng B. to recruit more C3b to bind to Ng C. to enhance Ng's uptake into the host cell D. to avoid the immune response by coating Ng with "self" antigens E. That's how the body recognizes that Ng is a pathogen

to inactivate C3b when it binds to Ng

What is the main function of TC cells in the immune system? A. to activate PMNs B. to activate TH cells C. to activate B cells D. to initiate the complement system E. to release perforin and granzymes

to release perforin and granzymes

Why do some viruses produce an analog of MHC1? A. To stop NK cells from killing cells infected by the virus B. So the viruses can bind to the MHC1 and be internalized more readily C. To be able to present viral proteins more easily on the surface of the host cell D. It is a viral toxin that initiates an autoimmune reaction in the host E. To reduce the efficiency of interferon binding to neighboring cells

to stop NK cells from killing cells infected by the virus

Why does a bacterium produce antioxidants? A. To help your body fight off infections from other bacteria. B. To signal phagocytes to engulf it. C. To survive the harsh environment inside the phagolysosome. D. The bacterium is producing its own chemoattractants. E. To kill the phagocyte that is trying to engulf it

to survive the harsh environment inside the phagolysosome

Nosocomial transmission of diseases is typically prevented by all of the following EXCEPT A. use of masks and gloves for all patients and health care workers B. cohorting of nurses and patients C. positive pressure in rooms, negative in halls and common areas D. use of ethanol-based hand sanitizers E. regularly changing catheters and I.V. lines

use of masks and gloves for all patients and health care workers

A newborn with a case of trachoma is an example of . . . A. Vertical disease transmission B. Horizontal disease transmission C. An immunodeficiency disease D. Seasonal disease incidence E. An endemic disease focus

vertical disease transmission

Which of the following is NOT associated with tuberculosis? A. bone and liver granulomas B. survival of the organism in macrophages C. vertical transmission D. lung necrosis E. long-term treatment with multiple antibiotics

vertical transmission

A wart such as the one shown at the right is caused by a . . . A. virus B. systemic fungus C. dermatophytic fungus D. spore-forming bacterium E. protozoan

virus

In the precipitin test shown by this picture, well C contains IgG against diphtheria toxin. Which of the following must be true? A. Well A contains diphtheria toxin. B. Well B contains diphtheria toxin. C. The white line between wells A and C contains aggregated clumps of cells. D. Fluorescent antibodies have been added to well C. E. This is a type of direct ELISA

well A contains diphtheria toxin

How do Treg cells differ from TH cells? A. When Treg cells bind B7, they engulf and destroy it. B. Treg cells secrete apoptotic cytokines that kill nearby cells. C. Treg cells phagocytose and digest pathogens in the gut. D. Treg cells recognize endogenous antigens on MHC-I. E. Treg cells recognize self antigens in addition to foreign ones

when Treg cells bind B7, they engulf and destroy it

Why does a second exposure to the same pathogen usually result in a much stronger immune response against it? A. Macrophages retain a memory of the pathogen and phagocytose it more effectively. B. Antibodies produced during the first infection can remain in the body for years. C. When a second infection occurs, Bmemory cells produce IgG without class switching. D. When a second infection occurs, T cells can be directly stimulated by macrophages without a B cell intermediate. E. During a second infection, Bmemory cells can be produced without TH cell involvement

when a second infection occurs, Bmemory cells produce IgG without class switching

What would be the most apparent way to distinguish rubeola from rubella? A. Rubella is a respiratory disease B. White spots are present in the lining of the mouth in cases of rubeola C. Rubella has a much more prominent rash D. Jaundice is present in cases of rubeola E. Patients with rubella have a much higher fever

white spots are present in the lining of the mouth in cases of rubeola

Which of the following is a major portal of entry, but not a major portal of exit? A. Upper Respiratory Tract B. Digestive Tract C. Wounds D. Vaginal Epithelial Cells E. Unbroken Skin

wounds

Can the structure shown at the right be toxic? If so, how? If not, why not? A. Yes, the part labeled 'A' can combine with antibodies to create a hypersensitivity reaction B. Yes, the part labeled 'B' can bind to a protein that induces a strong cytokine response from macrophages. C. Yes, the whole structure can bridge the interaction between naive TH cells and APCs D. No, the structure as shown lacks the toxic portion, and can only bind to receptors. E. No, the structure is not bacterial and has nothing to do with a toxin

yes, the part labeled 'B' can bind to a protein that induces a strong cytokine response from macrophages

You are testing antibiotics in a mouse model of a new bacterial disease. You have determined the MIC for the antibiotics from an E-test, and administered each antibiotic to several mice at greater than the MIC dose. But all the mice still die. Which of the following would be a logical conclusion from the results of this experiment? A. You should look for the production of a toxin by the new bacterium. B. You shouldn't have used an E-test to determine the MIC of the antibiotics. C. This new disease is probably polymicrobial. D. You should have administered the antibiotics at a dose less than the MIC, not more. E. Animal models are not valid for study of human diseases

you should look for the production of a toxin by the new bacterium

Histoplasmosis is very common in Indiana. Which of the following is the most likely way to be infected with this illness? A. Someone with Histoplasmosis coughs on you. B. You sit on a thorn that is contaminated with Histoplasma spores. C. You inhale dry dust that has been contaminated with rodent urine. D. You eat food that has been contaminated by Histoplasma bacteria. E. You tear down a shed that has a large accumulation of bird feces inside it.

you tear down a shed that has a large accumulation of bird feces inside it


Related study sets

Lab 7: Retina, Macula and Fovea, Optic Disc and Optic Nerve

View Set

Lifespan Chap 32: Concepts of Care for Patients with Cardiac Problems

View Set

Combo of NY BAR Review cards by smjr1920

View Set

Chapter 26: EMT: Behavioral Emergencies Post study Questions

View Set

Chapter 29 The High-Risk Newborn: Problems Related to Gestational Age and Development

View Set

Public Speaking DE - Ch 12 Using Language

View Set